Princeton Review Exams: Psych/Sociology

¡Supera tus tareas y exámenes ahora con Quizwiz!

Based on the information in the passage, the nucleus accumbens and ventral tegmental area are likely associated with which neurotransmitter system? A. Dopamine Correct Answer B. Serotonin C. Norepinephrine D. Acetylcholine

A. According to the passage, the nucleus accumbens and the ventral tegmental area are involved in reward processing; dopamine plays a major role in reward-motivated behavior (choice A is correct). While serotonin dysregulation is associated with depressive symptoms, serotonin is not involved in reward processing (choice B is wrong). Norepinephrine (also called noradrenaline) is involved in arousal, as well as in learning and mood regulation, and is not linked to reward processing (choice C is wrong). Acetylcholine is primarily used by neurons in the peripheral and central nervous systems to control functions ranging from muscle contraction and heart rate to digestion and memory (choice D is wrong).

An individual's description of her behavior at work earlier that day is an example of retrieval from: A. episodic memory. Correct Answer B. semantic memory. C. procedural memory. D. olfactory memory.

A. Episodic memory refers to memory of one's personal experiences (choice A is correct). Semantic memory is memory of concepts and information (choice B is wrong), while procedural memory is memory of how to execute motor tasks (choice C is wrong). Olfactory memory is not a term associated with models of long-term memory. In the context of short-term memory, it would refer to a sensory store from which one could recall what a given stimulus smelled like. It would not have relevance to recall of one's behavior (choice D is wrong).

Where is oxytocin produced and released, respectively? A. Hypothalamus, posterior pituitary gland Correct Answer B. Hypothalamus, adrenal gland C. Posterior pituitary gland, hypothalamus D. Posterior pituitary gland, amygdala

A. Oxytocin is produced in the hypothalamus (choice C and choice D are wrong). However, oxytocin is not secreted by the hypothalamus. Rather, the hypothalamus links the autonomic nervous system to the endocrine system via the pituitary gland, which is responsible for releasing the hormones oxytocin and vasopressin (choice A is correct). The adrenal gland is also responsible for the secretion of hormones, but is primarily responsible for the secretion of stress hormones such as cortisol, epinephrine, and norepinephrine, as well as some androgens (choice B is wrong). The amygdala is part of the limbic system and plays a role in the regulation of emotion.

Emotion plays a critical role in attitude change, and a great deal of attitude research notes the importance of affective/emotional components. Important factors to consider with regard to the impact of emotion on attitude are: self-efficacy, attitude accessibility, issue involvement, and message source/features. Of these factors, which has the greatest potential to change people's emotional self-perception (i.e., the way that individuals feel about themselves)? A. Self-efficacy Correct Answer B. Attitude accessibility C. Issue involvement D. Message source/features

A. Self-efficacy is the perception of human agency - the perception of one's ability to face a given situation. It dictates one's ability to deal with the emotion caused by a situation and to process that emotion via perception (choice A is correct). Attitude accessibility refers to the activation of an attitude from memory. Studies have shown that accessible attitudes are change-resistant (choice B is wrong). Issue involvement is the relevance and salience of an issue or situation to an individual; it is correlated with attitude access and attitude strength, both of which are distinct from perception (choice C is wrong). Message source and the features of the message being sent are on an object-subject correlative; they are not directly relevant to perception (choice D is wrong).

"T" is a fourteen-year-old high school freshman who last month joined the swim team at his school. Each morning before practice he is subjected to a ritual wherein the more senior members of the team insult and ridicule him, casting aspersions on his masculinity and athletic prowess. Humiliating and obscene slogans are often written on his locker, and his personal items are strewn around the school. Often these incidents lead to lengthy verbal altercations between T and the other team members. When T brings his situation to the attention of the swim coach, he is told that the older students are simply "testing him," and that the teasing will stop once T has proved himself to the team. When T speaks with the assistant principal in this regard, he is told that little can be done so long as the other students are not physically harming or touching him. The hazing incidents continue as before, although T no longer tries to defend himself or protect his property. T's change in behavior can most likely be attributed to: A. the fundamental attribution error. B. learned helplessness. Correct Answer C. internal locus of control. D. aversive conditioning.

B. Learned helplessness is a mental state in which the individual no longer tries to escape from a bad situation, or to otherwise help himself, because of past unsuccessful attempts to do so. T appears to have stopped fighting back because his failed attempts to get help have caused him to believe that resistance is futile (choice B is correct). The fundamental attribution error refers to the tendency to attribute others' behavior to dispositional factors, as opposed to situational ones. There is no indication in the question stem that T is attributing the behavior of the rest of the team to dispositional factors (choice A is wrong). Individuals with an internal locus of control tend to believe that they have the power to control what happens to them. The opposite appears to be the case for T, who now shows signs of an "external locus" belief system, thinking that he is powerless to control outside forces (choice C is wrong). Aversive conditioning entails attempting to eliminate unwanted behavior by pairing it with an unpleasant stimulus (e.g., pairing alcohol with nausea-inducing drugs). It is typically a voluntary therapeutic process and does not explain T's change in behavior in any way (choice D is wrong).

If a person went to a sold-out baseball game that was very noisy and was still able to hear her name yelled out from many seats away, this would be an example of: A. Treisman's attenuation model of selective attention. B. Broadbent's filter model of selective attention. C. the cocktail party effect. Correct Answer D. selective priming.

C. The cocktail party effect occurs when one is suddenly able to hear one's name, or another piece of information of personal importance, from previously unattended to channels (choice C is correct). Treisman's attenuation model tries to account for selective attention by explaining that the brain "turns down the volume" on unwanted or unnecessary sensory data. This would not explain why one would hear one's name in a crowded and noisy stadium (choice A is wrong). The Broadbent filter model suggests that unwanted or unnecessary sensory input is filtered out before it can be stored in short-term memory. This would not explain why one would hear one's name in a crowded stadium either (choice B is wrong). Selective priming occurs when one encounters particular sensory data very frequently or is expecting to encounter that particular data. While one does encounter one's name frequently, one would not expect to hear it at a stadium, so this choice does not fully explain why one would be able to pick out one's name in a noisy stadium (choice D is wrong).

What is the difference between prejudice and discrimination? A. Prejudice is a set of beliefs about a group of people, and discrimination is negative and judgmental attitudes based on those beliefs. B. Discrimination is a set of beliefs about a group of people, and prejudice is positive behavior based on those attitudes. C. Discrimination is negative and judgmental attitudes based on beliefs, and prejudice is behavior based on those attitudes. D. Prejudice is negative and judgmental attitudes based on beliefs, and discrimination is behavior based on those attitudes. Correct Answer

D. A set of beliefs about a group of people is a stereotype (choice A is wrong). Prejudice is an attitude based on stereotypes that includes emotions and value judgments as well. Prejudices are typically negatively valenced against a group of individuals, and do not include behaviors (choices B and C are wrong). Discrimination is behavior based on those attitudes (choice D is correct).

Given the information presented in Table 1, the data shown in Figure 1 suggest that: A. a mammal's relative olfactory bulb size is a reliable predictor of olfactory sensitivity to sulfur-containing compounds, but absolute size is not. B. a mammal's absolute olfactory bulb size is a reliable predictor of olfactory sensitivity to sulfur-containing compounds, but relative size is not. C. both absolute and relative olfactory bulb size are reliable predictors of a mammal's olfactory sensitivity to sulfur-containing compounds. D. neither absolute nor relative olfactory bulb size is a reliable predictor of a mammal's olfactory sensitivity to sulfur-containing compounds. Correct Answer

D. From Table 1 it is evident that the human olfactory bulb has a larger absolute size than the mouse olfactory bulb, while the mouse olfactory bulb has a larger relative size. If absolute size were a reliable predictor of olfactory sensitivity to a given odorant, then the percentage of human subjects who detected sulfur at a given threshold would have exceeded the percentage of mouse subjects that detected sulfur at that threshold across all three odorant conditions. In contrast, if relative size were a reliable predictor, then the percentage of mouse subjects that detected sulfur at a given threshold would have exceeded the percentage of human subjects who detected sulfur at that threshold across all three odorant conditions. Based on this information alone, it would be impossible for both relative and absolute size to be predictors of olfactory sensitivity (choice C is wrong). From the data depicted in Figure 1, however, it is apparent that a greater percentage of mice detected sulfur at <5 ppt in the ferret and bobcat conditions, but a greater percentage of humans detected sulfur at the same threshold in the weasel condition. Thus, neither absolute nor relative olfactory bulb size was a reliable predictor of olfactory sensitivity (choice D is correct; choices A and B are wrong).

Which of the following statements is true of sleep? A. Babies generally require less sleep than do adults. B. Older adults generally require more sleep than do children. C. Teenagers generally require more sleep than do babies. D. Older adults generally require less sleep than do younger adults. Correct Answer

D. The exact number of hours of sleep per night that each age group requires is debatable, but researchers do at least agree upon the general trends in sleep patterns. As adults grow older, they often require significantly less sleep than do their younger counterparts (choice D is correct). Babies tend to need much more sleep than adults do (choice A is wrong). As stated above, older adults generally need much less sleep than do younger populations, including children (choice B is wrong). Although teenagers often need more sleep than older adults do, babies tend to need more sleep than is needed by both teenagers and older adults (choice C is wrong).

Wernicke's area would be associated primarily with which of the following activities? A. Decoding the text B. Circling the ratings on the IITQ C. Asking a research assistant for help D. Comprehending the text Correct Answer

D. Wernicke's area is a region in the left hemisphere of the brain responsible for language comprehension (choice D is correct). It is not responsible for decoding, which involves pronouncing the sounds of letters and words (choice A is wrong), nor is it associated with motor activity (choice B is wrong). Broca's area, not Wernicke's area, is associated with language production (choice C is wrong).

A prison is an example of: A. a utilitarian organization. B. a coercive organization. Correct Answer C. a normative organization. D. a punitive organization.

B. A prison, wherein membership is involuntary, is an example of a coercive organization (choice B is correct). Utilitarian organizations are those that provide some sort of remuneration or benefit to members (choice A is wrong). Normative organizations involve voluntary membership based on shared goals or interests (choice C is wrong). A "punitive organization" is not one of the three formal organizations in sociology (choice D is wrong).

An affective disorder involving a pattern of comparatively mild depression that lasts for at least two years is known as: A. cyclothymic disorder. B. dysthymic disorder. Correct Answer C. seasonal affective disorder. D. bipolar disorder.

B. Dysthymic disorder, a less severe pattern of depression, is characterized by the sad mood, lack of interest, and loss of pleasure associated with major depression. These symptoms are milder but longer lasting. To qualify as a dysthymic disorder, the symptoms must last for at least two years in adults and one year in children (choice B is correct). Cyclothymic disorder is a milder form of bipolar disorder and is characterized by an alternating pattern of mood swings (choice A is wrong). Seasonal affective disorder is a calendar-linked pattern of depressive episodes, with depressive symptoms most prominent during months of shorter daylight (choice C is wrong). Bipolar disorder is an affective disorder characterized by alternating extremes of mood, from depression to mania and back again (choice D is wrong).

Which well-researched concept states that individuals are less likely to intervene to help someone in the presence of a large crowd? A. The social facilitation effect B. The bystander effect Correct Answer C. The Yerkes-Dodson law D. Altruism

B. The bystander effect states that, as the number of observers increases, the likelihood that someone in need will receive assistance decreases. The underlying theory states that people will diffuse the responsibility amongst themselves (i.e., they will assume that someone else will intervene) to the point where no one ultimately acts (choice B is correct). The social facilitation effect and the Yerkes-Dodson law both deal with how an individual's performance is affected by other people. The social facilitation effect states that people perform better on simple tasks when others are watching (choice A is wrong). The Yerkes-Dodson law refers to how stress and arousal affect performance; a moderate level of stress helps people perform at their best (choice C is wrong). Altruism refers to instances wherein an individual helps another despite incurring a cost or risk for helping. Large crowds of witnesses actually reduce the chance of altruistic behavior (choice D is wrong).

Because researchers chose to use a questionnaire, which of the following was missing from this study? A. Dependent variables B. Scientific method C. Independent variables D. A control group Correct Answer

D. The use of a questionnaire means that the researchers did not utilize a control group and thus did not have two groups participating simultaneously under different circumstances; this would simply not be possible with this methodology (choice D is correct). The dependent variables in this study are the preferences regarding age and gender of the students polled in the questionnaire—what the researchers were looking to study (choice A is wrong). The scientific method, in which hypotheses are empirically tested through the careful and systematic collection of relevant data. was used in this study and is ideally used in all scientific research (choice B is wrong). The independent variable here is the group of students polled (those between the ages of 18 and 30, identifying as either male or female; choice C is wrong).

Based on the passage, which of the following results might be expected if the researchers utilized a Korean sample and all participants received social support?

A. According to the passage, the behavioral phenotypes of the OXTRgenotypes in Eastern cultures are the reverse of what they are in Western cultures. Thus, individuals who exhibit the GG/AG genotypes in Western cultures likely have a similar behavioral phenotype to individuals who exhibit the AA genotype in Eastern cultures. Figure 1 shows that German individuals (i.e., individuals from a Western culture) with the GG/AG genotype who receive social support demonstrate reduced stress reactivity relative to German individuals with the AA genotype who receive social support. Therefore, in Eastern cultures, it is likely that individuals with the AA genotype will exhibit reduced stress reactivity following social support relative to individuals with the GG/AG genotypes (choice A is correct). Choice B demonstrates the predicted outcome for a Western culture, but the passage indicates that the predicted outcome for an Eastern culture would be the reverse (choice B is wrong). In choices C and D, there is no difference between genotypes. However, the passage indicates that we would expect that individuals with different genotypes would respond to stress following social support differently (choices C and D are wrong).

In the semantic network model, what determines the strength of a connection between a node and an association? A. How frequently and deeply connections are madeCorrect Answer B. How closely the node and association are explicitly related C. How many links there are between the node and the association Your Answer D. How many state-dependent cues have been created between the node and the association

A. According to the semantic network model, the strength of a connection between a node and an association is related to how frequently and how deeply the connections are made (choice A is correct). The explicit relationship between the node and the association is meaningless unless that connection is constantly reinforced (choice B is wrong). Separate links between the node and the association—suggesting a more circuitous route—are indicative of the spreading activation pattern (choice C is wrong). State-dependent cues, in which familiar locales are used to trigger memories, do not determine the strength of the connection between a node and an association unless, once again, they are used with great frequency (choice D is wrong).

What would Albert Bandura suggest as one of the reasons why the children whose mothers watched the videotaped training scored higher than the children in the control group and the children whose mothers were trained in the dialogic technique face-to-face? A. Those who trained with the videotape could model their behavior on the trainer from the tape. Correct Answer B. Those who trained with the videotape could watch the tape at home and thus were more relaxed and receptive to its teachings. C. Those who trained with the videotape could pause the tape when they wanted to take notes, a luxury denied people who are trained face-to-face in real time. D. Those who trained with the videotape preferred the direct instruction technique.

A. Albert Bandura proposed social learning theory, which includes learning via modeling: a person sees another person perform a behavior and then imitates that behavior. Having a single trainer on a videotape that could be played repeatedly gave mothers the opportunity to model the trainer's behavior and learn the dialogic reading strategies well via modeling. These mothers could then use the reading strategies properly and effectively with their children (choice A is correct). While it is possible that participants who watched the tape at home were more relaxed and receptive to its teachings, this does not best explain Albert Bandura's suggestions as to the reasons why the children whose mothers watched the videotaped training scored higher (choice B is wrong). Similarly, while it is possible that participants in the videotape condition may have been able to take notes, this does not best explain Albert Bandura's suggestions as to why the children in the videotape group scored higher either (choice C is wrong). According to the third paragraph of the passage, the mothers in the face-to-face training group received direct instruction, while the mothers in the videotaped training group received demonstrative training (choice D is wrong).

Conduction aphasia occurs when some sort of damage occurs to the connections between Wernicke's area and Broca's area in the brain. Although language comprehension and production are both individually intact, this disconnect leads to specific problems with: A. repeating information back to someone. Correct Answer B. reading printed words. C. writing. D. facial recognition.

A. Although a patient with conduction aphasia will understand what is being said to him or her and can communicate back, this disconnect caused by damage to the arcuate fasciculus makes it particularly hard to repeat aloud what has been said to him or her; the information cannot be relayed from Wernicke's area (processing) to Broca's area (production) properly (choice A is correct). Reading skills may still be intact, as the Wernicke's area is not damaged (choice B is wrong), and writing skills may also be intact, as Broca's area is not damaged (choice C is wrong). An inability to recognize faces, known as prosopagnosia, is due to damage to a certain visual pathway in the occipital and temporal lobes and is not related to language capabilities (choice D is wrong).

According to John Bowlby's attachment framework discussed in the passage, infants and human beings of all ages have an innate urge to: A. make strong emotional connections with others.Correct Answer B. maintain equilibrium between sexual and aggressive drives. C. use reason and discern meaning from their surroundings. D. seek safety and proximity to others.

A. Attachment theory, as developed by John Bowlby, is based on the belief that the inclination to make strong emotional bonds with particular individuals is a basic component of human nature that exists at birth and continues throughout life. Thus, human beings are motivated from birth onward to form connections with others. During childhood, bonds are formed with parents or primary caregivers, who provide protection, comfort, and assistance. During adolescence and adulthood, the bonds persist and are complemented by new bonds formed (choice A is correct). Ego psychology is a school of psychoanalysis rooted in Sigmund Freud's structural id-ego-superego model of the mind. Proponents of ego psychology focus on the ego's normal and pathological development, which involves its management or balance of sexual and aggressive impulses and drives (choice B is wrong). Rationalists, such as Plato, Kant, and Descartes, placed emphasis on the activity of the mind and the capacity to reason as being intuitive. Thus, rather than placing emphasis on relationships with others, information about the world, including relationships, is organized and understood based on reasoned discussion and debate (choice C is wrong). Seeking proximity and safety is a fundamental part of the attachment framework. The responsiveness of the parent to the child when he or she seeks proximity and safety dictates the style of attachment formed. However, the innate urge to make bonds and connections with others precedes the proximity-seeking and safety-seeking behaviors (choice D is wrong).

Suppose that, prior to receiving the text, participants in each condition were asked to rate their level of self-efficacy in reading comprehension. Which of the following results would the researchers likely expect? A. Incremental theorists would have higher self-efficacy ratings than would entity theorists. Correct Answer B. Entity theorists would have higher self-efficacy ratings than would incremental theorists. C. Both groups would have low self-efficacy ratings. D. Both groups would have high self-efficacy ratings.

A. Bandura's social cognitive notion of self-efficacy refers to the confidence people have in their ability to be successful at a given task. Given incremental theorists' belief in the malleability of intelligence and the effectiveness of increased effort (see solution to question 5), they would possess a greater degree of self-efficacy in the face of a difficult comprehension task (choice A is correct). Entity theorists, however, would see the greater difficulty as posing a potentially insurmountable challenge, so they would likely have lower self-efficacy ratings (choices B, C, and D are wrong).

Rates of ADHD diagnosis have increased approximately 5% per year from 2003 to 2011. What might a conflict theorist suggest about this? A. The rise in ADHD diagnoses made by physicians reflects a systemmatic attempt to secure more insurance money for childhood behavioral disorders. Correct Answer B. Since more males than females are diagnosed with ADHD, an increased rate of diagnosis reflects gender inequality in society. Your Answer C. This statistic is misleading; since rates of diagnosis for all psychological disorders have increased in the past few decades, the increased rates of ADHD diagnoses are really just reflective of a larger problem. D. The diagnostic criteria for ADHD have evolved to encompass more symptoms, thus leading to leading to the perception that more children than ever before are being disgnoses with the disorder.

A. Conflict theorists believe that society evolves through struggle or conflict between social structures, which are each trying to get the largest possible share of limited resources. Therefore, a conflict theorist would be most likely to suggest that the rise in ADHD diagnoses reflects a systematic attempt to secure more insurance money for childhood behavioral disorders, since this is the only choice that reflects an attempt to secure more of a limited resource (insurance money; choice A is correct). While true that more males than females are diagnosed with ADHD, an increased rate of diagnosis does not reflect gender inequality in society, as some psychological disorders are simply more common in one sex or the other; furthermore, issues of gender inequality would be more likely addressed by a feminist theorist than a conflict theorist (choice B is wrong). While true that diagnosis rates for most psychological disorders have increased in the past few decades, indicating a larger problem with the system, this is something that a social constructionist (not a conflict theorist) would most likely suggest, since this would indicate that society's definition of "mental illness," as a social construct, is shifting (choice C is wrong). A social constructionist (not a conflict theorist) might also suggest that diagnostic criteria for ADHD have evolved to encompass more symptoms, thus leading to the perception that more children than ever before are being diagnosed with the disorder; this would also reflect a shifting in our conception of what ADHD is as a social construct (choice D is wrong).

Psychologist Ernest Hilgard also replicated the ice bath study and found that, although participants reported that they felt no pain, when asked to hit a button if some part of them felt pain, many did. Which psychological phenomenon might explain this behavior? A. Dissociation Correct Answer B. Accommodation C. Conformity D. Extinction

A. Dissociation is defined as a split in consciousness, and this phenomenon can allow contradictory thoughts or behaviors to occur simultaneously. In the case of a person reporting no pain but indicating otherwise, dissociation might explain this behavior (choice A is correct). Accommodation is the process by which one adapts one's current understanding to incorporate new information; the stem does not provide any information to suggest that accommodation was relevant (choice B is wrong). Conformity refers to the phenomenon in which individuals adjust their behaviors or beliefs to coincide with those of the dominant group; there is no information in the stem to suggest that individuals were under pressure to conform (choice C is wrong). Extinction is a term used in classical and operant conditioning and refers to the diminishment of a conditioned association; there is no evidence of conditioning in the question stem (choice D is wrong).

H. M. was an American medical patient who suffered from severe seizures following a bicycle accident as a child. In 1953, he was treated by removing approximately 2/3 of his left and right hippocampi, as well as parts of other brain structures. He subsequently suffered from which disorder? A. Anterograde amnesia Correct Answer B. Epilepsy C. Schizophrenia D. Procedural amnesia

A. H. M. suffered from complete anterograde amnesia following the removal of most of his hippocampi, meaning that he could no longer form new episodic memories (choice A is correct). While H. M. did suffer from epilepsy, that existed prior to his surgery, and the seizures were largely eradicated by the treatment (choice B is wrong). Schizophrenia is believed to be an inherited disorder, although its expression is still impacted by the environment; H.M. did not exhibit schizophrenic symptoms (choice C is wrong). H. M. was able to acquire new skills, though he was unable to remember having learned them (choice D is wrong).

Much research demonstrates a close relationship between social connections and health outcomes. In the weeks leading up to a stressful exam, it can be hypothesized that: A. students who are lonely will have poorer immune responses than will those who are socially connected.Correct Answer B. students who are socially connected will have poorer immune responses than will those who are lonely. C. students' social networks will not influence their immune responses. D. students' decrease in social interactions will improve their immune responses.

A. High quality social relationships serve as a buffer against the deleterious health effects of stress. Therefore, it can be inferred that students without those relationships (i.e., those who are lonely) will be less protected from the negative health effects of stress (i.e., poor immune functioning; choice A is correct). By this reasoning, socially-connected individuals will be less likely to have poor immune responses than will lonely individuals, and decreasing social interactions might diminish one's immune response (choice B and choice D are wrong). Because the immune response is a health outcome that is affected by the presence of a social stressor, there does exist a relationship between students' social networks and their immune responses (choice C is wrong).

The hippocampus is generally responsible for which brain function? A. Memory Correct Answer B. Emotion C. Motor control D. Personality

A. The hippocampus is the structure of the brain that is responsible for memory (choice A is correct). Emotions are not produced by the hippocampus, although emotional memories are stored differently in the hippocampus than are non-emotional ones. Thus, while the hippocampus is part of the limbic system, it does not produce emotions (choice B is wrong). The part of the brain that controls motoric actions is the motor cortex, not the hippocampus (choice C is wrong). Personality is a complex set of behaviors and reactions that is generally thought to be centered in the frontal lobes, as evidenced by severe personality changes following injuries to that part of the brain (e.g., the case of Phineas Gage; choice D is wrong).

Which of the following could be the task that the participants were asked to perform in the control condition of this study? A. Participants are asked to think about a time when they were content and comfortable. They are asked to focus on that event with no instruction as to how to think about it.Correct Answer B. Participants are asked to think about a stressful event and to consider the causes, meanings, and consequences of that event. C. Participants are asked to think about a stressful event that has not happened yet, and to think about their apprehensions about it. D. Participants are asked to consider a time when they were unsuccessful and to attempt to figure out the reasons for the failure.

A. In experimental studies the control group does not receive the experimental treatment. The experimental conditions in this study are the rumination induction and the worry induction. The control group should experience a condition that does not induce worry or rumination, but still requires the participant to think about something. In choice A, participants are asked to think about a time when they were content and comfortable. They are asked to focus on that event with no instruction as to how to think about it. Under these circumstances there is no worry induction involving apprehension of a future event, nor is there rumination induction involving consequences of a past event (choice A is correct). In choice B, the participants are asked to think about a stressful event and to consider the causes, meanings, and consequences of that event. This would induce rumination by going over a past event repeatedly in order to try to understand it (choice B is wrong). In choice C, participants are asked to consider an event that has not yet occurred and any apprehensions they have about it. This would induce worry by focusing on the anxiety and apprehensions about a future event (choice C is wrong). In choice D, participants are asked to consider a time when they were unsuccessful and to attempt to figure out the reasons for the failure. Again, this would induce rumination by repeatedly going over a past event in an attempt to understand a problem (choice D is wrong).

After an individual has diverged from a social group, that group would be considered: I. the individual's out-group. II. the individual's in-group. III. the individual's reference group. A. I only Correct Answer B. II only C. III only D. I and II only

A. Item I is true: an out-group is defined as a social group with which the individual does not identify, which is the same concept as a dissimilar group from which someone would tend to diverge. After an individual diverges from a social group, that group would be considered his or her out-group (choice B can be eliminated). Item II is false: an in-group is defined as a social group with which the individual identifies; an individual might choose to diverge from a group because he or she does not want to be associated with that group or be mistaken for a member. After the person has diverged, that group would not be considered his or her in-group (choice D can be eliminated). Item III is false: reference groups are used as a way for individuals to compare themselves and their cultural tastes to a social group with which they identify. After an individual has diverged from a social group, that group would not be considered his or her reference group (choice C can be eliminated, choice A is correct).

Which of the following statements can be inferred about role strain for women? I. Women tend to be under more stress than men are in professional schools due to impression management. II. A female student in medical school is an example of front-stage self. III. A mother interacting with other mothers at the park is an example of back-stage self. A. I and II only Correct Answer B. I and III only C. II and III only D. I, II, and III

A. Item I is true: since role strain is conflict resulting from having multiple identities or from playing multiple roles, as female students in professional schools do, it is related to impression management. People use impression management in order to craft their front-stage selves, and the difficulty of doing so can cause stress known as role strain (choice C can be eliminated). Item II is true: a front-stage self as viewed from the dramaturgical perspective is the self that the individual "performs" in front of an audience. The female's role as a student in medical school is performed in front of other members of society such as the other medical students, teachers, and staff (choice B can be eliminated). Item III is false: a back-stage self, as viewed from the dramaturgical perspective, is the self that the individual is not performing in front of any audience. The woman in this scenario is still playing the role of a mother in front of other mothers, which is considered front-stage behavior (choice D can be eliminated and choice A is therefore correct).

Which of the following is/are an accurate description of a big five personality trait? I. Extraversion: People high in extraversion tend to be outgoing, sociable, upbeat, and friendly. II. Conscientiousness: People high in conscientiousness tend to be curious, flexible, and imaginative. III. Kindness: People high in kindness tend to be sympathetic, trusting, and cooperative. A. I only Correct Answer B. II only C. I and II only D. I and III only

A. Item I is true: the big five traits are extraversion, neuroticism, openness to experience, agreeableness, and conscientiousness. Extraversion is one of the big five traits, and people high in extraversion tend to be characterized as outgoing, sociable, upbeat, and friendly (choice B can be eliminated). Item II is false: conscientiousness is one of the big five traits, but this trait is associated with being diligent, disciplined, and well-organized. It is not associated with curiosity, flexibility, or imaginativeness (choice C can be eliminated). Item III is false: kindness is not one of the big five traits (choice C can be eliminated and choice A is correct).

It is possible that gentrification contributes to residential segregation of blacks and whites at the expense of minorities through which of the following mechanisms? I. This process restructures local neighborhoods such that the new structure better supports those with higher incomes. II. This process causes rising housing prices such that long-term residents cannot manage the costs. III. This process increases the tax base used to provide essential services in white neighborhoods, but not in black neighborhoods. A. I and II only Correct Answer B. I and III only C. II and III only D. I, II, and III

A. Item I is true: there is an important socioeconomic component to both gentrification and residential segregation. The process of gentrification tends to benefit the higher-income newcomers who are responsible for the urban renewal, which can then contribute to the displacement of the lower-income native residents of the urban neighborhood. There is a significant relationship between socioeconomic status (SES) and race in the United States; minorities tend to be SES-disadvantaged compared to whites (choice C can be eliminated). This is fundamental to the concept of gentrification; residential shifts most often have negative effects for native residents of color and positive effects for new higher-income whites. Item II is true: rising housing prices is one of the main results of the process described in the explanation of the first item (choice B can be eliminated). Item III is false: there are economic advantages associated with gentrification as discussed in the passage. This, however, does not explain the mechanism through which gentrification contributes to residential segregation (choice D can be eliminated and choice A is the correct answer). The increase in residents with higher incomes in urban populations does contribute to the tax base of the affected cities, but the consequences are not necessarily limited to the higher income neighborhoods. The higher tax base in these cities should be distributed so as to provide needed services in each neighborhood wherein taxes are paid.

How might a theorist in the tradition of symbolic interactionism explain deviance? A. Deviant behavior is a response to being labeled a deviant by others. Correct Answer B. Deviant behavior results from performing on the back stage when one should be performing on the front stage. C. Deviance is a failure to develop a looking-glass self. D. Deviance is a failure to resolve the dilemma of identity versus role confusion in adolescence.

A. Labeling theory is a fundamental concept of symbolic interactionism. It states that deviance is not inherent in the act itself, but is rather an attribution from others (such as police, family, or clergy; choice A is correct). Back stage and front stage dynamics are an aspect of the dramaturgical approach. If one acts on the back stage when he or she should be performing on the front stage, the audience might experience more than it otherwise would (e.g., more details about a person's motivations or inner thoughts), but this would not necessarily be a deviant act (choice B is wrong). A looking-glass self describes how people shape their self-concepts based on how they think others perceive them (choice C is wrong). The dilemma of identity versus role confusion is one of Erikson's eight developmental stages; it is not an aspect of symbolic interactionism (choice D is wrong).

According to Lawrence Kohlberg, what is the highest level of morality that the average adult tends to achieve? A. She or he will acknowledge a sense of duty to uphold the laws of the land and maintain basic social conventions.Correct Answer B. He or she will strive for social justice—the greatest good for the greatest number. C. He or she will focus on the approval (or disapproval) of others, acting in accordance with the views of his or her community or peer group. D. She or he will act in such a way as to avoid punishment at all costs.

A. Lawrence Kohlberg postulated six stages of moral development, but said that the average person generally does not pass beyond the fourth stage, in which he or she acknowledges a sense of duty to uphold the law and maintain social conventions (choice A is correct). Striving for social justice, wherein one acts for the greatest good for the greatest number, is the fifth stage in Kohlberg's schema and one that he proposed that few people reach (choice B is wrong). A focus on receiving approval from one's peers along with a need to conform is the third stage of development, usually seen in older children and adolescents (choice C is wrong). Acting to avoid punishment is the earliest stage of moral development, typical of very young children (choice D is wrong).

Which of the following is a negative symptom of schizophrenia? A. Avolition Correct Answer B. Disorganized speech C. Hallucinations D. Delusions

A. Negative symptoms of schizophrenia are defined as deficits in normal behavior, such as avolition (the lack of motivation to initiate behavior). Someone experiencing avolition might remain in the same position for long periods unless moved by someone else (choice A is correct). Positive symptoms of schizophrenia are those that most individuals do not experience but which are present in people with schizophrenia. Disorganized speech lacks coherence and a goal; this is a positive symptom (choice B is wrong). A hallucination is the perception of sensation (feeling, hearing, or seeing) in the absence of an external stimulus (e.g., hearing a voice when no one is speaking or seeing something that no one else sees); this is a positive symptom (choice C is wrong). Delusions are false beliefs that are held with conviction despite contradictory evidence; this is also a positive symptom of schizophrenia (choice D is wrong).

The difference between normative social influence and informational social influence is: A. normative social influence is about people conforming for social acceptance, and informational social influence is about people conforming for "the right reasons" and because "others know more". Correct Answer B. normative social influence is about people conforming for "the right reasons" and because "others know more", and informational social influence is about people conforming for social acceptance. C. normative social influence is about people conforming to help normalize society, and informational social influence is about people conforming in the interests of educating society. D. normative social influence occurs when people decide for themselves to conform, and informational social influence occurs when an authority figure, such as a political or religious leader, motivates people to conform.

A. Normative social influence occurs when people conform because they desire social acceptance. In contrast, informational social influence occurs when people conform because they want to make ethically correct choices and also because they believe that others know more than they do (choice A is correct). Normative social influence does not occur out of a desire to be morally correct or to follow those more knowledgeable, nor does informational social influence occur when people desire social acceptance (choice B is wrong). Normative social influence has nothing to do with normalizing society or establishing social norms. Similarly, informational social influence has nothing to do with informing or educating society (choice C is wrong). Normative social influence is not about people deciding for themselves when to conform. However, informational social influence could occur when an authority figure tells people to conform because the people could decide either that the authority figure is morally correct or that he or she knows more than they do (choice D is wrong).

Media representations of Planned Parenthood often center on their abortion practices. Planned Parenthood is responsible for an annual total of 300,000 abortions. However, this accounts for a mere three percent of the services provided. This misrepresentation of Planned Parenthood is an example of: A. institutional discrimination. Correct Answer B. the McDonaldization of society. C. a coercive organization. D. symbolic interactionism. Your Answer

A. Planned Parenthood experiences the effects of prejudice as a result of practices, such as abortion, that conflict with the beliefs of some groups, and this prejudice, in some cases, contributes to institutional discrimination. Mass media is a powerful institution in the United States, and the media executives with higher socioeconomic status are able to set the agenda for their programs. This can lead to institutional discrimination in situations when the media presents a specific group in a negative light for its own purposes (e.g., the anti-abortion movement; choice A is correct). The McDonaldization of society described when a culture adopts the characteristics of a fast-food restaurant, such as efficiency, calculability, standardization, and control. While aspects of mass media are certainly a good example of McDonaldization, the misrepresentation of Planned Parenthood by the media is not an example of this (choice B is wrong). A coercive organization is one where members have no choice in joining; for example, a prison is an example of a coercive organization. While Planned Parenthood has little choice in what the media portrays about them, as an organization, Planned Parenthood is not a part of the media - these are two separate organizations. Therefore, this is not an example of a coercive organization (choice C is wrong). Symbolic interactionism is a major sociological theory that suggests that people create symbolic meanings about objects, events, and behaviors. For example, a handshake is a symbol in society that conveys many different meanings—and everyone in society pretty much acts toward this symbol in agreed-upon ways. Symbolic interactionism is a micro-level theory of society, which focuses on interactions between individuals. Therefore, even if the interaction between the media and Planned Parenthood neatly conveyed something symbolic, it still could not be an example of symbolic interactionism because these are two social structures, which is a macro-level view of society (choice D is wrong).

In a study of word association, participants were given a prompt word and asked to decide if the next word they were given was an English word or a nonsense word. The results of the study showed that participants recognized words more quickly when there was a semantic relationship to the prompt word. For example, participants given the prompt word TEACHER more quickly recognized STUDENT as an English word than MUONAS as a non-English word. This type of semantic word association is a form of implicit memory known as: A. priming. Correct Answer B. recognition. C. recall. D. conceptual salience.

A. Priming is a phenomenon wherein one stimulus implicitly affects the response to another stimulus; it is a type of implicit memory. Priming can happen across semantic categories (as highlighted in the question stem), as well as across perceptual and conceptual categories (choice A is correct). Recognition is a type of memory recall in which a person recognizes having seen or heard a word before; the scenario in the question stem highlights a process of semantic association, not recognition (choice B is wrong). Recall is a general term for remembering an item that one previously heard or saw (choice C is wrong). Conceptual salience refers to the prominence of a word or object in one's mental representations, but it does not deal with the type of semantic association shown in the question stem (choice D is wrong).

What aspect of the dialogic reading method described above best aligns with B. F. Skinner's model of language acquisition? A. Repeating children's correct answers and then encouraging and praising them Correct Answer B. Asking "what" and other open-ended questions C. Modeling good answers for the children D. Making reading into a game so that children have fun

A. Repeating children's correct answers and then encouraging and praising them is part of Skinner's model of language acquisition by operant conditioning. He believed that language use, like any other behavior, is subject to conditioning and requires repetition and positive reinforcement (choice A is correct). Asking open-ended questions, part of this dialogic method, does not best align with Skinner's behavioral methodology (choice B is wrong). Modeling good answers for the children would be more in keeping with Bandura's theory of modeling than with Skinner's idea of conditioning (choice C is wrong). Making the activity fun is simply part of the dialogic method to keep children engaged and motivated. It is not best aligned with Skinner's theory (choice D is wrong).

As a first-year medical student, RJ feels pressure to study very hard and do well in his classes, but also feels pressure not to look like he is "trying too hard" to do well. These competing expectations can best be described as: A. role strain. Correct Answer B. role conflict. C. social facilitation. D. stereotype threat.

A. Role strain occurs when there are competing expectations for the same status. In the scenario provided, RJ is dealing with the competing expectations to perform well, but also to not look like he is trying too hard, for the same status—that of first-year medical student (choice A is correct). Role conflict occurs when two statuses have competing expectations; since there is no indication that RJ feels conflict from the competing expectations of more than one status, role conflict is not occurring here (choice B is wrong). Social facilitation occurs when an individual is able to perform simple tasks better in front of an audience; this is not relevant to RJ's scenario (choice C is wrong). Stereotype threat occurs when an individual experiences a self-fulfilling fear that he or she will confirm a negative stereotype.There is no indication of any negative stereotypes here, nor does the scenario suggest that RJ's performance is adversely affected by anything. Therefore, stereotype threat does not apply (choice D is wrong).

Suppose that most people believe that male students are more likely to excel in medical school than are female students. Which of the following would illustrate stereotype threat for this scenario? A. Female students producing low exam scores due to anxieties about performing poorly Correct Answer B. Male students producing high exam scores due to anxieties about performing poorly C. A majority of women not applying to medical school because they are not interested in the science field D. A majority of men applying to medical school because they are pressured by others to pursue careers in medicine

A. Stereotype threat occurs when an individual's performance is adversely affected by feelings of anxiety about being labeled with a negative stereotype. The female students mentioned performed poorly on exams due to anxiety about the negative stereotype (choice A is correct). The male students did not perform poorly because of a negative stereotype; in fact, the reverse happened (choice B is wrong). Women who do not choose to be in medical school because of their lack of interest are not responding to stereotype threat (choice C is wrong). Men who apply to medical school because of pressure from others are responding to social pressure rather than to stereotype threat (choice D is wrong).

Suppose that a public school teacher begins to focus on challenging only the middle-class students academically because she expects them to excel more than the lower-class students. Noticing this, lower-class students begin to worry about being negatively stereotyped; as a result, 94% of middle-class students excel (receiving grades of 85% or higher) while 97% of lower-class students underperform (receiving grades or 75% or lower). Which of the following concepts most accurately describes these results? A. Stereotype threat Correct Answer B. Social stigma C. Ethnocentrism D. Attribution theory

A. Stereotype threat occurs when an individual's performance is adversely affected due to feelings of apprehensiveness about confirming a negative stereotype with which he or she has been labeled. The question stem states that the students worried about confirming the teacher's stereotype of lower-class students, which resulted in underperformance (choice A is correct). Social stigma is a negative reaction that others have towards an individual because he or she does not fit the social norms of society Although lower-class students are subject to stigma based on their economic background, this does not explain the underperformance of the lower-class students as well as stereotype threat does (choice B is wrong). Ethnocentrism is the belief that one's ethnic group is superior to others; the scenario describes a belief based on class rather than ethnicity (choice C is wrong). Attribution theory involves attempts to explain the underlying causes of the behaviors and actions of others; the question stem does not describe a scenario in which anyone tries to find the cause of another's actions (choice D is wrong).

Which of the following sociological theories suggests that society is composed of groups or institutions that share common norms, have a definitive culture, and work together to maintain overall societal equilibrium, and which sociologist is most closely associated with this theory? A. Structural functionalism; Emile Durkheim Correct Answer B. Structural functionalism; Max Weber C. Symbolic interactionism; Max Weber D. Symbolic interactionism; Emile Durkheim

A. Structural functionalism is the sociological theory that suggests that society is a complex system whose parts work together to promote and maintain overall societal equilibrium; this theory views society at the macro-level, and broadly focuses on the social structures that shape society as a whole. The theorist most closely associated with structural functionalism is Emile Durkheim (choice B is wrong and choice A is correct). Symbolic interactionism asserts that people act toward things, such as objects, events, and behaviors, based on the meaning that these things have for them and that these meanings are derived from social interaction and modified through interpretation. This theory is a micro-level theory of society, and focuses on individual interactions rather than on broader social structures. Symbolic interactionism is most closely associated with two key theorists: George Herbert Mead and Charles Horton Cooley (choices C and D are wrong). Max Weber is most often associated with conflict theory.

Suppose Subject 405 returned home after the experiment and watched a television program about mammalian predators. Before a commercial break, the program's narrator explained that bears secrete kairomones and challenged viewers to think of the primary chemical contained in bear kairomone. During the break subject 405 confidently told her roommate that "no doubt the primary chemical is sulfur!" Assuming that subject 405 has no familiarity with kairomones other than through her experience as a study participant, her assertion exemplifies which of the following? A. The availability heuristic Correct Answer B. The optimistic bias C. The representativeness heuristic D. The central route to persuasion

A. The availability heuristic refers to people's tendency to solve problems or estimate probabilities based on information that is most readily available, or accessible, to their minds. Thus, Subject 405's assertion reflects her recent experience as a participant in the study rather than a logical approach toward problem solving (choice A is correct). The optimistic bias is people's tendency to assess the probability of personal success in future endeavors with more optimism than is warranted by actual facts (choice B is wrong). The representativeness heuristic refers to a problem-solving strategy by which people make choices based on their perception of which option is most characteristic of the issue in question. Since the question stem assumes that Subject 405 has no familiarity with kairomones other than through her participation in the study, there is no basis for Subject 405 to consider sulfur to be more representative of mammalian kairomones than any other substance (choice C is wrong). When people use logic or information-based arguments to persuade others, they are considered to be using the central route of persuasion. Subject 405, however, did not offer any such arguments to substantiate her claim (choice D is wrong).

Current research on overall job satisfaction (Figure 1) and overall job-related stress (Figure 2) among physician specialists demonstrates substantial differences among the specialties. In both figures, the five specialties with the highest and lowest reports are represented, as well as the mean response. Which of the following can be most reasonably concluded from these data? A. There is some evidence of a negative association between job satisfaction and job-related stress. Correct Answer B. There is some evidence of a positive association between job satisfaction and job-related stress. C. There is conclusive evidence of an association between job satisfaction and job-related stress. D. There is no evidence of an association between job satisfaction and job-related stress. Your Answer

A. The data presented in the figures suggest a possible negative relationship between job satisfaction and job-related stress (choice D is wrong). There is some evidence that specialists reporting lower job-related stress (e.g. preventative medicine, general pediatrics, and dermatology) also report higher job satisfaction (choice A is correct; choice B is wrong). However, the data provided are certainly not conclusive (choice C is wrong). For example, the opposite does not appear to be true; the three specialties reporting the lowest rates of job satisfaction are not the three specialties reporting the highest rates of job-related stress. It is difficult to provide conclusive statements about the relationship between these two factors with the limited data presented; the figures provided represent only the five specialties at either extreme of the two spectrums.

If the results shown in Figure 1 ran contrary to the evolutionary theory mentioned in the passage and showed a more egalitarian society, what might the data demonstrate? A. More men would actually choose women closer to their own age or even older. Correct Answer B. The gap in age difference for the actual partners for men and women would both widen, with men choosing even younger women and women choosing even older men. C. There would be less of a gap in the age differences between men's actual and ideal partners. D. There would be less of a gap in the age differences between women's actual and ideal partners.

A. The evolutionary theory is based on the idea of perpetuating the species and thus focuses on men's choice of women who are younger and seemingly more fertile and who would enable them to reproduce. Thus, if men chose women who were their own age or older, it would run contrary to the evolutionary theory (choice A is correct). A widening of the gap in the ages of actual partners, with men choosing even younger women and women choosing even older men, would actually strengthen the evolutionary theory. The theory states that women are primarily looking for a stable male to provide them and their children with sustenance (choice B is wrong). Lessening the gap between the actual and ideal age differences, for either men or women, would neither confirm nor contradict the evolutionary theory (choices C and D are wrong).

Which one of the following regions of the brain is most likely to be implicated in the onset and degenerative progression of Alzheimer's disease? A. Hippocampus Correct Answer B. Thalamus C. Posterior occipital lobe D. Superior parietal lobe

A. The hippocampus is associated with the formation, storage, and retrieval of memories. Since Alzheimer's often profoundly affects a sufferer's ability to remember, this region is most likely to be implicated in the onset and degenerative progression of AD (choice A is correct). The thalamus is a relay center for sensory information that is important for sensation and the perception of external stimuli. Since the primary symptom of Alzheimer's is memory loss, this is not the brain region most likely to be implicated (choice B is wrong). The posterior occipital lobe is critical to vision and the organization of visual information (choice C is wrong). The superior parietal lobe has many functions, including integration of spatial information and maintenance of internal representations; however, none of these functions involves the primary symptoms of AD (choice D is wrong).

The design of the study mentioned in the passage is experimental. What is the independent variable that was manipulated? A. The type of induction exercise Correct Answer B. The type of psychopathology C. Level of anxiety D. Level of depression

A. The independent variable in an experimental study is the variable that is manipulated by the researchers. By randomly selecting the individuals to participate in either the worry induction, the rumination induction, or the control condition, the researchers are controlling the environmental stimuli in terms of the type of induction exercise in which the subjects participate (choice A is correct). There is no way for the researchers to manipulate the type of psychopathology that the individual participants experience. The researchers simply found subjects for their study that had either been diagnosed with GAD or depression (choice B is wrong). The level of anxiety was the outcome measure being studied, and is therefore the dependent variable of this experiment (choice C is wrong). Level of depression was not examined in this study (choice D is wrong).

The levels of processing theory would be supported by a finding that: A. participants in the low coherence condition performed better on a task assessing long-term memory of the material than did participants in the high coherence condition, regardless of intelligence-theory orientation. Correct Answer B. participants in the high coherence condition performed better on a task assessing long-term memory of the material than did participants in the low coherence condition, regardless of intelligence-theory orientation. C. entity theorists in the high coherence condition performed better than incremental theorists in the same condition on a problem-solving task. D. incremental theorists in the high coherence condition showed lower emotional arousal in response to the text than did entity theorists in the same condition.

A. The levels of processing theory posits that people remember information better when they process it more deeply. For example, people who are made to answer questions about material they read will remember the material better than those who simply read it. Since participants in the low coherence condition must think more deeply about the text in order to understand it, it follows that they will remember the material better and thus perform better on a long-term memory task. Additionally, while the researchers expected differences in perceived comprehension between the two types of theorists, no expected differences in depth of processing or actual comprehension is indicated by the passage (choice A is correct; choice B is wrong). The passage did not suggest that entity and incremental theorists would differ regarding problem-solving ability (choice C is wrong) or emotional arousal (choice D is wrong). Further, neither problem solving nor emotional arousal is relevant to the levels of processing theory.

According to Figure 1, which of the following is NOT true about the participants who took the junk food home? A. Participants were less likely to take junk food in a public setting if they had recently read an article associating junk food with undergraduate students, as opposed to high school dropouts. Correct Answer B. Participants were less likely to take junk food in a public setting if they had recently read an article associating junk food with high school dropouts, as opposed to undergraduate students. C. Participants were less likely to take junk food in a private setting if they had recently read an article associating junk food with undergraduate students, as opposed to high school dropouts. D. Dissimilar others did not affect the participants' choices in the private setting, as it did in the public setting.

A. The passage indicates that undergraduate students are considered a similar group, and high school dropouts are considered a dissimilar group, by the undergraduate student participants. Figure 1 shows that there is a much higher number of participants who chose junk food in a public setting for similar groups rather than dissimilar groups (choice A is false and therefore the correct answer). Figure 1 shows that participants in the public setting picked junk food less often when it was associated with a dissimilar group (choice B is true and can be eliminated). According to Figure 1, more people chose junk food in the dissimilar group than in the similar group in the private setting condition (choice C is true and can be eliminated). Unlike the public setting condition, results showed that the private setting condition had a higher number of people who still chose junk food even if it was associated with the high school dropouts (choice D is true and can be eliminated).

Alzheimer's disease, at all stages, is known to affect cognitive centers of the brain. Which of the following is NOT typically an early symptom of Alzheimer's? A. Sleep apnea Correct Answer B. Forgetfulness C. Losing or misplacing personal items D. Trouble with simple problem-solving tasks

A. The question stem states that Alzheimer's disease affects cognitive centers. Sleep apnea is a somatic condition in which breathing is temporarily disrupted during sleep; it is not a cognitive problem (choice A is not a symptom of Alzheimer's disease and is therefore correct). Pronounced forgetfulness is one of the first detectable symptoms of Alzheimer's disease and indicates impairment of the cognitive centers. For example, unlike the more commonplace forgetfulness that is often seen in older individuals, a man experiencing the early symptoms of Alzheimer's might forget the name of his grandchild or the state in which he lives (choice B is an Alzheimer's symptom and is therefore wrong). A tendency to frequently lose or misplace personal items is also an early symptom of Alzheimer's disease and suggests disrupted cognition (e.g., putting laundry into the kitchen cabinet; choice C is an Alzheimer's symptom and is therefore wrong). Alzheimer's disease also initially manifests itself in terms of an impaired ability to carry out simple tasks and to solve everyday problems. For example. a woman might find herself unable to follow the steps of a recipe that she has successfully followed many times before (choice D is also an Alzheimer's symptom and is therefore wrong).

A group of researchers hypothesize that labeling young children "learning disabled" causes them to internalize a negative concept and produces harmful iatrogenic deficits in adolescence and adulthood. Which of the following would be the most appropriate design method to assess the accuracy of this hypothesis? A. Cross-sectional B. Longitudinal Correct Answer C. Experimental D. Case study

B. A longitudinal design is a type of observational study that involves collecting data from a group of individuals over a long period. It can be used to assess the effects of treatment or intervention across lifespans and would be appropriate in this case (choice B is correct). A cross-sectional design is a type of observational study that involves collecting data from a group of individuals at one particular point in time. It does not measure long-term effects or changes (choice A is wrong). In contrast to observational studies, an experimental design involves a manipulation of some sort. In this case, the researchers would have to label at least some children "learning disabled" and then evalute the effects of that action. However, if this manipulation is likely to be harmful to those children, which is what the researchers believe, then they cannot ethically conduct the experiment (choice C is wrong). A case study is an in-depth analysis of a single individual. In this case, it would not be appropriate or reasonable to attempt to generalize one child's experience of having received a diagnostic label to the experiences of such children in general (choice D is wrong).

According to Figure 2, which of the following statements is FALSE? A. Blacks and whites reported nearly identical limitations in activity. B. There was a greater difference in activity limitation reports between the racial groups than there was within the racial groups. Correct Answer C. A higher percentage of blacks than whites in all ranges of household income reported their health to be poor. D. Whites above the national poverty level had a lower number of poor health reports than did blacks above the national poverty level.

B. According to Figure 2, there was a much greater difference within, rather than between, the two racial groups under the "percent reporting activity limitations" section. In other words, whites below the national poverty level reported more activity limitations than did those above the national poverty level, and blacks below the national poverty level reported more activity limitations than did those above the national poverty level, but there was little difference between blacks and whites within each poverty level category (choice B is false and is therefore the correct answer). The "percent reporting activity limitations" section shows that blacks and whites have roughly the same percentages for each poverty level (choice A is true and can be eliminated). According to the "percent reporting poor health" section, each percentage in the black column is higher than the corresponding percentage in the white column (choice C is true and can be eliminated). The table shows that 6.8% of non-poor whites reported poor health in the United States, which is the lowest percentage in the "percent reporting poor health" section (choice D is true and can be eliminated).

Which of the following wavelengths of light would be most easily detected at an angular distance of 30 degrees from the fovea? A. Violet light of 420 nm B. Blue light of 470 nm Correct Answer C. Green light of 540 nm D. Yellow-green light of 570 nm

B. At 30 degrees from the fovea, Figure 1 shows that the concentration of cones is near a minimum, while the concentration of rods is near a maximum. Also, according to the passage, vision in the periphery is predominantly handled by rods. The passage further states that rods have peak absorption at wavelength of 498 nm; this is closest to blue light of 470 nm (choice B is correct). The three types of cones have peak absorptions near 420 nm, 540 nm, and 570 nm (choice A, choice C, and choice D are wrong). Cone concentration is low at the visual periphery relative to rods, so S, M, and L cone cells would not be as sensitive to light in this region as would rods.

If a participant in the study described above rated her probability of getting arthritis at 25%, how would she respond when informed that the probability of developing arthritis is 14%? A. She would likely only slightly revise downward her chances of developing the disease. B. She would likely significantly revise downward her chances of developing the disease. Correct Answer C. She would likely only slightly revise upward her chances of developing the disease. D. She would likely significantly revise upward her chances of developing the disease.

B. Based on the information presented in the last paragraph of the passage, people will significantly respond to positive feedback with regard to probability estimates. In other words, the hypothetical participant would probably significantly revise downward the estimate of developing arthritis, as the information presented (14% chance) is positive news relative to her original estimate of a 25% chance (choice B is correct and choice A is wrong). People respond to negative feedback with regard to probability estimates by only modestly altering their original estimates. However, since the hypothetical participant received positive feedback (the difference was much more in favor of not developing the disease), she is not likely to upwardly revise her original estimate at all (choices C and D are wrong).

"J" is a heterosexual woman in her mid-twenties. She is generally good about practicing safer sex with new partners, but sometimes she skips using a condom. When J's friend asked her why she does not always use a condom, J replied that she is not worried because she can get antibiotics at her university's student health center. When the friend pointed out that HIV is an STI that can't be cured with antibiotics, J replied "yeah, but HIV is just something that gay men get. I mean, I know I could get it, but I probably won't." What psychological or social theory best explains why J is not worried about contracting HIV? A. Social stigma Your Answer B. Cognitive dissonance Correct Answer C. Fatalism D. Peer pressure

B. Cognitive dissonance involves the notion that individuals experience discomfort when they harbor inconsistent beliefs, attitudes, or thoughts, or when their behavior conflicts with these cognitions. In order to reduce the resulting mental tension, the person may change the cognitions or change the behavior. This phenomenon can be observed among people who engage in risky conduct despite knowing that it is dangerous and attempt to reduce the dissonance that they experience. A heterosexual woman who does not engage in safer sex may downplay her HIV risk by relying on the stereotype that HIV is most prevalent in the gay male community (choice B is correct). Although social stigmas regarding homosexual sex do exist, they would not explain why J is not worried about contracting HIV (choice A is wrong). Fatalism is the belief that human beings are subject to fate or to an inevitable future; there is no indication that J believes that her HIV status is in any way predetermined (choice C is wrong). There is nothing to suggest that J's peers pressured her into disregarding her risk of contracting HIV; furthermore, J's friend could arguably use peer pressure to influence J to consistently use condoms (choice D is wrong).

Attachment theory suggests that once an attachment style is established, it tends to persist into adulthood and old age. Which of the following interactions is LEAST likely to be affected by a person's established attachment style? A. The interaction between a father and his seven-year-old son on the child's first day at a new school. B. The interaction between a teenager and a cashier at the local supermarket. Correct Answer C. The interaction between a middle-aged woman and her boss of fifteen years. D. The interaction between a newly wed husband and wife.

B. Established attachment styles have been shown to persist into adulthood and old age. This framework affects bonds formed with romantic partners and significant others (i.e., people who play an important role in one's daily life). It can be inferred that the interaction between a teenager and a cashier at the local supermarket is not one that is of great significance in the teenager's daily life (choice B is the interaction that is least likely to be affected by an established attachment style and is therefore correct). The interaction between a father and his seven-year-old son on the child's first day of school would be influenced by both of their attachment styles. The child's attachment style would affect his comfort level and ability to separate from his father in order to navigate and interact while at his new school. Similarly, the father's attachment style would affect his ability to let his child face this new stage of life without experiencing a great deal of anxiety himself (choice A is wrong). The interaction between a middle-aged woman and her boss of fifteen years would also be affected by the attachment styles of both individuals; they are significant people in each other's lives due to the long-term relationship and bond formed throughout the fifteen years (choice C is wrong). The interaction between newlyweds would also be affected by the attachment styles of both individuals, as their relationship will now be one of the most important and significant in their lives (choice D is wrong).

Which of the following statements about socioeconomic status is NOT supported by the passage? A. Michigan families earned more than American families earned in 2005, on average, but earned less in 2010. B. Detroit families earned more on average from 2000 to 2005, but earned less from 2005 to 2010. Correct Answer C. American families continued to earn more on average from 2000 to 2010. D. Detroit families earned less than all of the other reported families, on average, in 2000, 2005, and 2010.

B. Figure 1 presents detailed information about income, which is used as a common measure of socioeconomic status. Michigan families earned $57,300 in 2005 and $56,100 in 2010. U.S. families earned $55,800 in 2005 and $60,600 in 2010. Therefore, Michigan families earned more than American families earned in 2005, on average, but less in 2010 (choice A is true and can be eliminated). Detroit families, on average, earned $33,900 in 2000, $33,600 in 2005, and $31,000 in 2010. Therefore, Detroit families earned less in each period from 2000 to 2010 (choice B is not supported and is the correct answer). U.S. families earned $50,000 in 2000, $55,800 in 2005, and $60,600 in 2010. Therefore, U.S. families earned more, on average, in each period from 2000 to 2010 (choice C is true and can be eliminated). Detroit families, on average, earned less than families did in Michigan and in the U.S. in general in each period from 2000 to 2010 (choice D is true and can be eliminated). This is noticeable in Figure 1 because the differences are quite large. For example, the greatest difference, the income difference between U.S. families and Detroit families in 2010 - $60,600 for the former and $31,000 for the latter - is a highly significant $29,600. This has serious implications for those in Detroit, as their average earnings are around half the average earnings of families in the United States.

One aspect of socialization is learning the patterns of normal behavior of one's social group. Certain norms are formal and strongly enforced, while others are merely customary ways of behaving that don't carry harsh penalties when violated. These customary ways of behaving are known as: A. mores. B. folkways. Correct Answer C. values. D. sanctions.

B. Folkways are conventional ways of acting within a certain culture. Violating them would make one look unusual, but the violation would not carry an official penalty (choice B is correct). Mores, in contrast, are formal norms that are very strictly enforced because they protect basic tenets of society (e.g., treason violates mores of loyalty and patriotism; choice A is wrong). Values are a culture's standard for evaluating what is good and bad (choice C is wrong). Sanctions are the punishments for violating a norm (choice D is wrong).

"B" and "D" are the newest members of an elite military unit stationed overseas in hostile territory, having joined the group three weeks ago. Since they arrived, both have been subjected to nightly rituals of progressively severe physical and psychological abuse by the other soldiers as part of an ongoing "initiation rite." In response to a complaint filed by another soldier, the commanding officer asks both women whether they had been victims of any abuse. Both B and D emphatically deny that they suffered any abuse since joining the unit, even when offered an immediate transfer to another military base, and characterize any physical contact that may have occurred as good-natured fun. Moreover, they both speak well of their unit comrades, with the exception of the complaining soldier, whom they bitterly denounce as a liar. Which one of the following theories is LEAST helpful in explaining the actions of B and D? A. Peer pressure B. Groupthink Correct Answer C. Identification with the aggressor D. Cognitive dissonance Your Answer

B. Groupthink involves faulty decision-making in situations involving group interaction; there is no evidence of any such group decision-making here (choice B is least helpful in explaining these actions and is the correct choice). Peer pressure to remain silent and go along with the abuse helps explain the actions of B and D (choice A is helpful in explaining these actions and can be eliminated). The possibility that B and D may have identified with their aggressors, which is more apt to occur in situations of prolonged abuse in circumstances resembling captivity, is likely as well. The women are in military service and thus could not simply leave the situation when the attacks started. Moreover, the abuse appears to have been ongoing and severe for a substantial length of time. By the time an offer of transfer was made they may already have identified with their attackers; protecting and uniting with those whom they view as more powerful than themselves may be an attempt to regain self-esteem lost from continual humiliation and victimization (choice C is helpful in explaining these actions and can be eliminated). Anxiety associated with cognitive dissonance occurs when an individual holds contradictory or otherwise incompatible beliefs or attitudes. For B and D, a self-image as an elite and powerful soldier may be incompatible with a self-image as a helpless and desperate victim of abuse. Accordingly, they resolve the dissonance by denying that abuse occurred (choice D is helpful in explaining these actions and can be eliminated).

If the critics of the "lost in the mall" study are correct in their assessment (as stated in paragraph 3), this would suggest that the experiment lacks: A. internal validity. B. external validity. Correct Answer C. test-retest reliability. D. replicability.

B. Internal validity refers to the extent to which a causal conclusion based upon an experiment is warranted; a study that lacks systematic error is said to have internal validity. In contrast, external validity refers to the extent to which the results of an experiment can be generalized to other situations and to other people. In this case, the critics mentioned in paragraph 3 do not suggest that false memories of being lost in the mall were not generated under these experimental conditions (choice A is wrong). Rather, they suggest that false memories of trauma or abuse could not be similarly created, thus questioning the ability to generalize the study's findings to other situations and to other people (choice B is correct). Test-retest reliability applies to test scores and assesses the degree to which they are consistent from one administration to the next (choice C is wrong). Replicability refers to the ability to obtain the same results under the same experimental conditions. The critics mentioned in the passage do not contend that the study could not be replicated in its current form, but simply that the results do not generalize to other types of false memories (choice D is wrong).

How would Erik Erikson most likely respond to this study? A. Erikson would wholeheartedly agree with the premise of this study, as he believed that individuals do not develop their full identities until they reach adulthood, usually between the ages of 18 and 35. B. Erikson would challenge the premise of this study, as he believed that identity formation is a question that adolescents deal with primarily during the ages of 12 to 18. Correct Answer C. Erikson would agree with the premise of this study but challenge its results; he believed that one starts formulating an identity during the adolescent years, but it is not until the later years—middle age and later in life —that one can really "know" oneself fully. Your Answer D. Erikson would challenge the premise of this study, as he believed that identity formation is heavily influenced by unconscious factors.

B. In Erik Erikson's proposed stages of development, identity formation occurs during the adolescent years (ages 12 to 18). Consequently, Erikson would have most likely challenged the premise of this study, which implies that identity formation primarily occurs as adolescents transition to young adults (ages 18 to 22; choice B is correct). According to Erikson's developmental stages, when humans are between the ages of 18 and 35, they are mostly concerned with creating intimate relationships, rather than creating their own individual identities (choice A is wrong). As noted above, Erikson would disagree with the premise of this study. He would also say that by the time individuals reach middle age or the later years, they are less concerned with individual identity than they are with generativity vs. stagnation in middle age and integrity in the later years. By that point identity is really a non-issue unless it plays a role in either generativity or integrity (choice C is wrong). While Erikson believed in psychoanalytic notions regarding personality development, Freud was more focused on the unconscious, while Erikson believed that social factors influenced development over a lifespan (choice D is wrong).

A child who learns to avoid approaching her parent for help and comfort because she is always met with aggression by the parent develops an anxious-avoidant insecure attachment style. The child thus associates the parent with fear and begins to actively avoid interactions with the parent in order to avoid this emotional experience. This type of learning can also be explained by which operant conditioning principle? A. Modeling B. Avoidance learning Correct Answer C. Vicarious learning D. Role-playing

B. In avoidance learning, an organism's behavior is reinforced by the termination or prevention of an aversive stimulus. In other words, the individual is motivated to escape fear-producing environmental stimuli or conditioned stimuli. In this instance, because the child is always met with anger when she encounters the parent, the parent becomes a conditioned stimulus that produces fear. The child thus begins to actively avoid interactions with the parent in an effort to avoid experiencing fear (choice B is correct). Observational learning, also known as modeling or vicarious learning, is part of Albert Bandura's social learning theory. Bandura believed that learning could occur by watching others. However, not all observed behaviors are effectively learned. In order for the modeling process to work, according to Bandura, the following factors have to be present: attention, retention, reproduction, and motivation. The child in question is avoiding the parent and does not appear to be motivated to reproduce the aggressive behavior that she is observing (choices A and C are wrong). Role-playing is not an operant conditioning principle. Rather, it is an instance or situation in which one deliberately acts out or assumes a particular character or role. In psychology, it is a therapeutic technique designed to reduce conflict in social situations; participants act out particular behavioral roles in order to expand their awareness of differing points of view. Role-playing has also been the focus of many social psychology studies (choice D is wrong).

In the training stage for the mice, the dispensing of sugar water was intended to serve as: A. a negative reinforcer. B. a positive reinforcer. Correct Answer C. a conditioned response. D. an unconditioned stimulus.

B. In operant learning theory, the event that is perceived by the subject as desirable and which follows the target behavior is called a positive reinforcer (choice B is correct). The training stage of this study featured an operant design intended to reward the mice with sugar water for the target behavior of licking the metal dispenser whenever sulfur was detected. Negative reinforcement refers to the removal of a negative (or undesirable) stimulus in response to the target behavior (choice A is wrong). In classical learning theory, an unconditioned stimulus is a stimulus that naturally produces a particular reaction in the subject, which is called an unconditioned response (choice D is wrong). When an unconditioned stimulus is repeatedly paired with a neutral stimulus, the neutral stimulus becomes a conditioned stimulus, and the unconditioned response is made in response to the conditioned stimulus as well. At that point, the response is referred to as a conditioned response (choice C is wrong).

A researcher wants to challenge the hypothesis that OXTRpolymorphism genotype moderates the effect of social support on stress reactivity. The researcher proposes that individuals with GG genotypes were likely raised by parents with either GG or AG genotypes, and that the effect is actually environmental. What is one way that this researcher might test her hypothesis? A. Twin study B. Adoption study Correct Answer C. Transgenesis D. Developmental study

B. In order to test her hypothesis, the researcher would need to separate the influences of nature (genes) and nurture (the environment). An adoption study is the best way to do this. In an adoption study, researchers compare individuals with their biological and adoptive relatives in order to determine if they are more similar to one group or the other. If the researcher found that the effect shown in Figure 1 only persisted when individuals with the GG/AG genotypes were also raised by adoptive parents with GG/AG genotypes, this would suggest that the effect is due to environmental, rather than genetic, factors. However, if the researcher found the effect shown in Figure 1 for all adopted individuals, regardless of their adoptive parents' genotypes, this would be evidence for a genetic effect. Accordingly, an adoption study would be the best way to test this hypothesis (choice B is correct). In a twin study, researchers assess whether monozygotic (identical) twins are more similar than dizygotic (fraternal) twins in order to determine the extent to which a behavior can be attributed to shared genes, as opposed to a shared environment (choice A is wrong). Transgenesis involves either introducing a new gene or removing a gene from an animal's pre-existing genome in a tightly controlled experiment in order to assess the effect of that gene. Due to ethical concerns, it is highly unlikely that a human being would ever be a participant in a transgenesis study (choice C is wrong). A developmental study would allow the researcher to assess the effect of genotype on behavior at many different points throughout a person's life span; however, it would not allow her to assess whether an effect was due entirely or partially to genetic influence (choice D is wrong).

When a corporation chooses the location of new corporate facilities, which will provide employment opportunities for the surrounding community, based on the racial composition of the local neighborhoods and stereotypes about various minority groups, this corporation is demonstrating: A. individual discrimination. B. institutional discrimination. Correct Answer C. structural discrimination. D. reverse discrimination.

B. Institutional discrimination is the unjust treatment of a particular group by societal institutions; the passage mentions that some corporations intentionally factor racial composition and stereotypes into work decisions (choice B is correct). Unlike institutional discrimination, individual discrimination is the unjust treatment of a particular group by individuals (choice A is wrong). Structural discrimination is similar to institutional discrimination; however, structural discrimination occurs when individuals who run the institutions in question do not intend to treat any group differently or unjustly, but inadvertently do so through their established practices (choice C is wrong). Reverse discrimination is the unjust treatment of the dominant group; in the scenario presented, it is the minority group that is being negatively affected in terms of employment opportunities (choice D is wrong).

A finding that a mouse alarm pheromone shares molecular similarity with a bobcat's sulfuric odorant: I. confirms the researchers' first hypothesis. II. supports the researchers' second hypothesis. III. is consistent with the data in Figure 1. A. I only B. III only Correct Answer C. I and III only D. II and III only

B. Item I is false: a finding that an alarm pheromone in mice is molecularly similar to a bobcat's sulfuric odorant is consistent with the hypothesis that mice are, for reasons of survival, more sensitive to sulfur-containing odorants than are humans. However, this information is certainly not enough to confirm that hypothesis, which is in fact inconsistent with the overall results of the study. A pheromone is a chemical signal that an animal uses to communicate with members of its own species, such as those emitted by a queen bee to communicate with worker bees in the hive. Thus, a mouse alarm pheromone is a signal emitted by a mouse to alert other mice to the presence of a predator. While it is true that a greater percentage of the mice were sensitive to the bobcat kairomone, a greater percentage of the human subjects were sensitive to the weasel kairomone. Thus, the hypothesis that mice would exhibit more sensitivity (low detection threshold) than human beings would across all three kairomone conditions was not supported by the data overall (choices A and C can be eliminated). Item II is false: the molecular similarity of an alarm pheromone to a predator's kairomone does not provide any additional information regarding the relevance of olfactory bulb size to olfactory sensitivity to a given chemical (choice D can be eliminated). Item III is true: if a predator's kairomone odorant were molecularly similar to the mouse alarm pheromone, then one would expect mice to have a low olfactory detection threshold for that kairomone so that they could interpret the chemical signal in a manner similar to the way in which they interpret the alarm pheromone. The data in Figure 1 indicate that 100% of the mice detected sulfuric bobcat kairomone at a low threshold of < 5 ppt. Thus, a finding that a mouse alarm pheromone shares molecular similarity with the bobcat kairomone is consistent with the data (choice B is correct).

The results in Figure 1 suggest which of the following? Individuals with GG/AG genotypes demonstrate reduced stress reactivity Social support influences stress reactivity in individuals with GG/AG genotypes Stress is greater in individuals with the AA genotype who receive social support than in individuals with the AA genotype who do not A. I only Your Answer B. II only Correct Answer C. I and III only D. II and III only

B. Item I is false: not all individuals with GG/AG genotypes demonstrated reduced stress reactivity (choices A and C can be eliminated). Item II is true: Figure 1 indicates that individuals with the GG/AG genotypes demonstrate reduced stress reactivity following social support (relative to individuals with the GG/AG genotypes who do not receive social support and to individuals with the AA genotype). Item III is false: although Figure 1 demonstrates that social support moderates the association between genotype and stress reactivity, it only does so for individuals with a G allele (choice D can be eliminated and choice B is therefore correct).

Researchers used a probability-based sample to recruit non-physician participants, which lead to a non-physician sample representative of the national population. Therefore, even though demographic information is not presented in the passage, it is still possible to conclude that: I. the most common religious affiliation reported is Christian (as compared with Muslim, Jewish, Hindu, and Buddhist). II. the least common socioeconomic status reported is lower class (as compared with upper class, middle class, and working class). III. for married participants, the most common relationship structure reported is monogamous (as compared with polygamous).

B. Item I is true: the most common religion in the United States is Christianity (choice C can be eliminated). Christianity is the largest religion in the world, and in the United States 80% of the people identify as Christian. Item II is false: the least common socioeconomic status in the United States is upper class, not lower class (choices A and D can be eliminated). In the United States, the top 3% of the population is considered upper class, while roughly a quarter of the population is considered lower class. Item III is true: the most common form of marriage in the United States is monogamy. The social norm in the U.S. is monogamous marriages, in which the partners are married only to each other. Furthermore, this convention is reinforced through legislation, as polygamous marriages are illegal in the United States (choice B is correct).

Suppose that Subject 67 (quoted in the passage) is at a party for gay men and overhears a group of other men, including the party's host, having a conversation about him. The host mentions that Subject 67 is bisexual and indicates that this would be a disadvantage in terms of dating him, and almost all of the men in the group readily agree. One of the group members states that he wouldn't mind dating Subject 67 despite his bisexuality, but another member quickly rejects this statement and changes the subject. This type of censorship within a group in order to avoid disharmony is known as: A. mindguarding. Correct Answer B. social comparison. C. informational influence. D. group polarization. Your Answer

B. Mindguarding is an aspect of groupthink, which is the phenomenon wherein group members censor dissenting opinions in order to rmaintain group harmony and avoid "rocking the boat". In a groupthink situation, "mindguards" serve to shelter the group from controversial or potentially divisive information (choice A is correct). Social comparison is the process of evaluating one's opinion with regard to the opinion of another (choice B is wrong). Informational influence is similar to groupthink in that ideas that come into group discussion are those that tend to favor the dominant viewpoint. This, however, does not describe the deliberate censorship that this group member is engaging in so as to maintain group unity (choice C is wrong). Group polarization is a process by which the existing views of a group are strengthened during group discussion (choice D is wrong).

Research has also considered the salaries of medical professionals and the effects of compensation on overall life satisfaction. Reports of average earnings range from $156,000 to $315,000, with pediatricians reporting the lowest salaries and orthopedic surgeons and radiologists reporting the highest salaries. Based on this information, models of social stratification in the United States would consider the average anesthesiologist to be a member of which social class? A. Upper class Your Answer B. Middle class Correct Answer C. Working class D. Lower class

B. Most sociologists consider socioeconomic status (SES) when discussing social stratification. This measure is often defined in terms of power, prestige, and wealth owing to the common interrelationship among these factors. The reported salaries of specialists range from $157,000 to $316,000 for the lowest and highest earners listed, respectively. The average anesthesiologist is then expected to earn an intermediate amount (the research reports an annual average of approximately $310,000). While estimates differ, this earning potential alone suggests that anesthesiologists are middle class, perhaps upper middle class (choice B is correct). Middle class annual incomes range from $46,000 to the minimum earnings representing the upper class (this definition varies); 40% of the population is considered middle class. Despite the high earnings of these medical specialists in comparison to the national average, it is a common misconception that these professionals are "upper class". According to most models of social stratification, the income range for upper class is much higher. Upper class annual incomes range from millions to billions of dollars; a mere 3% of the population is considered upper class (choice A is wrong). There are medical professionals who earn comparable amounts when accounting for additional income and wealth (e.g., famous doctors who host their own medically-themed television shows can earn several million dollars each year). However, the information provided considers the average anesthesiologist. Working class annual incomes range from $19,000 to $45,000; 30% of the population is considered working class (choice C is wrong). Lower class annual incomes range from no income to $18,000; 27% of the population is considered lower class (choice D is wrong).

Nicotine belongs to which class of psychoactive drugs? A. Opiates B. Stimulants Correct Answer C. Depressants D. Hallucinogens

B. Nicotine is a stimulant; its overall effect is to speed up bodily functions (choice B is correct). Opiates belong to the class of psychoactive drugs called depressants. These work by slowing down bodily activity (choices A and C are wrong). Hallucinogens cause perceptual distortions, such as seeing colors that are not present in the environment or feeling moods of euphoria or terror (choice D is wrong).

According to Noam Chomsky, children learn all their vocabulary: A. solely from universal grammar. B. from a complex interaction of genetic and environmental factors. Correct Answer C. solely from environmental factors, such as parents and teachers reading to them from an early age. D. from a genetic predisposition to good reading and vocabulary skills. Your Answer

B. Noam Chomsky believed that a complex interaction of genetic and environmental factors is involved in building children's vocabularies (choice B is correct). However, he believed that the ability to learn language itself is innate (an idea that was eventually referred to as "universal grammar," a feature unique to humans that allows them to develop language skills but is not predictive of vocabulary; choice A is wrong). While Chomsky believed that environmental factors such as good schooling (good teachers and good educational programs) are important in developing children's vocabularies, he also believed that genetic factors play a role in vocabulary development and growth, since not all children in a class have equal vocabulary development (choice C is wrong). Conversely, Chomsky believed that genetic factors alone are insufficient to account for a child's vocabulary strength (choice D is wrong).

Nativists contend that language will be acquired by all normally developing humans, assuming they are exposed to language: A. after the post-conventional stage. B. before the critical period. Correct Answer C. along with positive reinforcement. D. through conditioning.

B. Noam Chomsky is the main theorest associated with the nativist perspective of language development, which contends that all normally functioning humans who are exposed to language before a critical period will develop language (choice B is correct). The post-conventional stage is the last stage of Lawrence Kohlberg's theory of moral development; it has nothing to do with language acquisition (choice A is wrong). The nativist approach is in direct opposition to the behaviorist model (based on Skinner's operant conditioning approach to learning), which suggests that language is acquired through conditioning using reinforcement and punishment; the nativist approach suggests that conditioning has nothing to do with language acquisition (choices C and D are wrong).

This study was repeated with a pool of subjects who were assessed as either high or low in neuroticism. Each of the following results is consistent with the outcome of the study described in the passage EXCEPT: A. both high and low neuroticism children tended to display the most hostility in the same situation. B. children high in neuroticism displayed little hostility, and displayed less hostility than usual in situations wherein low neuroticism children displayed more hostility than usual.Correct Answer C. the level of hostility displayed by high neuroticism children varied across situations. D. when averaging the levels of hostility displayed across situations, the high neuroticism children were more hostile than were low neuroticism children.

B. People who score high in neuroticism tend to be anxious and hostile. The above study suggests that the level of hostility displayed by a child will not only depend on his or her level of neuroticism, but also on the situation. The study does not support a finding that high neuroticism children and low neuroticism children have opposite reactions to a given situation (choice B is correct). The study is consistent with a finding that a given situation causes the most hostility for all children (choice A can be eliminated). The study shows that children's behavior is highly variable across situations. Hence, it is consistent with a finding that high neuroticism children showed varying levels of hostility across situations (choice C can be eliminated). The stability of situation-behavior profiles of children in the study in the passage is consistent with a finding that high neuroticism children, on average, show more hostility than do low neuroticism children (choice D can be eliminated).

The psychiatric term for a person's "loss of contact with reality" is: A. splitting. B. psychosis. Correct Answer C. defense mechanism. D. paranoia.

B. Psychosis is properly defined as a "loss of contact with reality" (choice B is correct). Splitting is a term used to describe "all or nothing" thinking, in which a person vacillates between thinking of the positive and negative aspects of situations or people, but cannot bring them into a nuanced whole. Psychiatrists consider this a defense mechanism, and it is a trait associated with borderline personality disorder (choice A is wrong). A defense mechanism is an unconscious coping strategy that a person uses to deal with a stressful situation (choice C is wrong). Paranoia is a condition characterized by anxiety, suspicions of conspiracy, perceived threats, and distrust of others (choice D is wrong).

Based on the information provided in the first paragraph, the interactional strategy that S is using while talking to her friend about why she does not have children is best described as: A. passing. B. substituting one stigmatized identity (voluntary childlessness) for a less stigmatized identity (infertility).Correct Answer C. back-stage behavior. D. repression.

B. S's utterance described at the end of the first paragraph is deliberately ambiguous; the phrase "not in the cards" could mean that S has tried and failed to have children, but it could also mean something else. Because the statement is ambiguous, S's friend might conclude that S is infertile. S thereby substitutes a less stigmatized identity for a more stigmatized one, which best explains S's interactional strategy (choice B is correct). "Passing" refers to attempts to disguise one's stigmatized identity. This would involve using interactional strategies that match those of parents, such as talking about desired parenthood. S does not desire parenthood, and her statement does not imply that she is attempting to pass (choice A is wrong). Back-stage behavior is an aspect of Goffman's dramaturgical theory of interaction. It describes the behavior and manner of a person when he or she is not performing any role in front of others; back-stage behavior occurs in private. S is not in private and is interacting with others; furthermore, she is caught in a situation wherein she must play a complicated ambiguous role on the front stage of the interaction with her friend (choice C is wrong). Repression is the act of expelling urges and desires from one's conscious mind into one's unconscious. There is not enough information in the passage to conclude that S's interactional strategy has anything to do with repression (choice D is wrong).

"M" likes and respects her therapist, who has stated repeatedly that M suffers from dissociative identity disorder (DID) and likely has several alternate personalities, although M remains skeptical. When M concedes that a particular memory lapse "could possibly have been due to the appearance of an alternate personality," her therapist praises her for being brave enough to confront the painful reality of her condition. When asked to imagine what one of her alternates might be like, M describes a six-year-old girl, and her therapist praises her as before. During a subsequent session M allows her therapist to "speak with" this girl, after which the therapist praises her profusely. M now believes that she suffers from DID. The process by which M came to accept the DID diagnosis might be explained as: A. social learning therapy. B. shaping. Correct Answer C. negative reinforcement. D. aversive conditioning.

B. Shaping is an operant conditioning technique that involves producing a desired behavior (in this case acceptance of the DID diagnosis) by reinforcing successive approximations of that behavior (choice B is correct). Social learning therapy involves an attempt to change an individual's behavior by having him or her observe a model being reinforced for exhibiting desirable responses (choice A is wrong). Negative reinforcement is an operant conditioning technique in which the frequency of a desired behavior is increased by following that behavior with the removal of an unpleasant or undesirable stimulus. Praise from her therapist would be a desirable stimulus for M (choice C is wrong). Aversive conditioning is a behavior modification technique that attempts to eliminate an unwanted behavior by pairing it with an unpleasant stimulus (e.g., administering a mild electric shock each time the subject takes a sip of an alcoholic beverage; choice D is wrong).

Which psychological theory of intervention is most closely associated with hypnosis? A. Attachment theory B. Psychoanalytic theory Correct Answer C. Social learning theory D. Humanistic theory

B. Sigmund Freud's psychoanalytic theory is most closely associated with the use of hypnosis, due to its focus on the unconscious. In psychoanalytic theory, hypnosis is used to access the client's unconscious desires, while "protecting" them from conscious thoughts and defense mechanisms (choice B is correct). Attachment theory is not typically associated with the use of hypnosis as an intervention, and instead focuses on increasing the responsiveness and sensitivity of a child's caregiver, rather than delving into the unconscious of the child (choice A is wrong). While social learning theory may explain why some individuals are more susceptible to hypnosis than others, in and of itself, the theory is not closely associated with hypnosis as an intervention, and instead focuses on deconditioning or reconditioning maladaptive behaviors and replacing them with more adaptive behaviors (choice C is wrong). Humanistic theory is not at all associated with hypnosis, and instead focuses more on self-help and Gestalt techniques to induce change in the individual (choice D is wrong).

Based on Study 2, which sociological concept is most relevant to an explanation of the difference in behavior exhibited by participants in public versus private settings? A. Bystander effect B. Impression management Correct Answer C. Multiculturalism D. Deindividuation

B. Study 2 examines the participants' behaviors based on the concept of the public and private self; impression management involves the idea that individuals employ both a "front stage" and "back stage" self when interacting with others, which is demonstrated by the difference in the amount of junk food selected when participants were in the public vs. private conditions (choice B is correct). The bystander effect is the phenomenon in which the presence of other people impedes an individual's likelihood of helping someone else during an emergency; this theory is unrelated to Study 2 (choice A is wrong). Multiculturalism is the promotion of multiple cultures living harmoniously within one community; this concept is also unrelated to Study 2 (choice C is wrong). Deindividuation is the loss of self-awareness in social groups; though social groups had an impact on participants' choices, deindividuation does not best explain the results of Study 2 (choice D is wrong).

All of the following could trigger aggression, but which one is most closely associated with the concept of observational learning? A. Aversive stimuli, such as extreme temperatures B. Watching videos or movies with violent contentCorrect Answer C. Neuropsychiatric diseases, such as dementia D. Aggressive behavior used for the purpose of self-protection in threatening situations

B. The bobo doll experiment conducted by Alfred Bandura (1961) found that children who observed an aggressive adult model tended to show more aggressive behavior themselves, as compared to children who were exposed to a nonaggressive adult model. Accordingly, watching videos or movies with violent content could trigger aggression, and this is the only answer choice that is associated with observational learning (choice B is correct). Aversive stimuli, such as extremely cold or hot temperatures, may lead to tenseness and anger that might cause increased aggression (e.g., in many major cities the crime rate does tend to increase during the hottest summer months). However, aggressive responses triggered by aversive stimuli are not learned through observation (choice A is wrong). Aggressive behavior may occur because of neuropsychiatric diseases such as dementia, which is characterized by the frontal disinhibition of the brain. Those diseases are caused by various factors but are not based on learning concepts (choice C is wrong). Perceived threats may trigger aggressive acts. Self-protection in situations perceived to be threatening is based on a present potential for physical or mental aggression, but is not related to the concept of observational learning (choice D is wrong).

The use of a "token economy" described in the last paragraph suggests that: A. ADHD is a strictly behavioral phenomenon that can be addressed through reward and punishment. B. children with ADHD respond to token economies better than do children with other psychiatric disorders. C. symptoms of ADHD can be reduced by applying principles of operant conditioning. Correct Answer D. the academic performance of those with ADHD is positively correlated with their socioeconomic status.

C. A token economy is a simple system of rewards for desirable behavior, which is one type of operant conditioning. By referring to a token economy as a component of standard intervention, the passage suggests that such a system can be used by parents to reduce problematic behaviors associated with ADHD (choice C is correct). While choice A accurately refers to a token economy as a reward system, it also refers to ADHD as a "strictly behavioral phenomenon". This characterization is erroneous, as some ADHD symptoms pertain to cognitive processes. Further, the fact that the disorder can be mitigated by medication implies a neurophysiological basis (choice A is wrong). Nowhere does the passage contrast the effectiveness of token economies regarding other disorders with their effectiveness in the context of ADHD (choice B is wrong). Additionally, the term "token economy" is not related to the term "economy" in the conventional sense, and so it is thus unrelated to any issue of socioeconomic status (choice D is wrong).

Which of the following scenarios illustrates the fundamental attribution error in the case of "K", a student with ADHD? A. K receives an "A" on her test. Her friend remarks, "I studied hard for my 'A', but K was just lucky." B. K turns to talk to her friend while the teacher is speaking. After class, the teacher comments to a colleague, "That Kimberly is a rude person. It's just her nature to be nasty!"Correct Answer C. K fidgets with her pencil during class, and her teacher thinks to herself, "K sure is irritating me, but I know that her ADHD is making it difficult to sit still." D. K starts daydreaming during class. Her teacher thinks, "K must be distracted by the sound of the lawnmowers outside."

B. The fundamental attribution error refers to the tendency of people to attribute the negative behaviors of others to their core personalities, rather than to situational or circumstantial factors. In K's case, her ADHD represents a circumstantial factor that contributes to her behavior. If her teacher attributes K's negative behavior to K's character or personality rather than to complications of her disorder, she has committed the fundamental attribution error (choice B is correct). If, however, she attributes the behavior to a circumstantial factor rather than to a dispositional one, she has not committed the fundamental attribution error (choice C is wrong) - even if she attributes the behavior to the wrong external factor (choice D is wrong). If K's classmate attributes her own success to personal achievement and K's success to circumstance, she has committed other types of faulty attribution, namely the self-serving bias and the actor-observer effect (choice A is wrong).

Which one of the following brain structures is most likely to be the site of false memory implantation? A. The hypothalamus B. The hippocampus Correct Answer C. The parietal lobe D. The cerebellum

B. The hippocampus is largely associated with memory processes and would be the likely site of implantation of false memories (choice B is correct). The hypothalamus is associated with autonomic nervous system functions and hormonal processes (choice A is wrong). The parietal lobe is associated with processing sensory information pertaining to taste, temperature, and touch (choice C is wrong). The cerebellum is associated with the regulation and coordination of movement, posture, and balance (choice D is wrong).

Do the results of the study presented in the passage support the information processing theory? A. Yes, because the individuals with GAD are more likely to experience anxiety through rumination, while the individuals with depression are more likely to experience anxiety through worry. B. Yes, because the individuals are engaging in cognitive processing through the use of cognitive strategies that are responses to environmental stimuli. Correct Answer C. No, because the researchers were not able to evaluate the types of cognitions produced by the inductions. D. No, because the individuals are not engaging in cognitive processing in response to environmental stimuli.

B. The information processing theory states that human beings process the information that they receive with their cognitions, rather than merely responding behaviorally to stimuli or internal drives. In the study above, the researchers are purposefully inducing different types of cognitions that are eliciting responses from the individuals. It can be clearly seen from Figure 1 that individuals in the worry induction and rumination induction conditions react differently than do those in the control condition. It can therefore be concluded that subjects are using their cognitions and cognitive strategies (though maladaptive) to interact with the environmental stimuli presented in the research conditions (choice B is correct). Choice A is incorrect because the graph shows that individuals with GAD are more likely to experience anxiety rumination (choice A is wrong). The researchers did evaluate the types of cognitions experienced by the participants during the inductions (choice C is wrong). Further, the participants were in fact engaging in cognitive processing in response to environmental stimuli (choice D is wrong).

It can be inferred from the passage that: A. socioeconomic status is correlated with positive Alzheimer's disease outcomes. B. learning complex tasks may prevent Alzheimer's symptoms. Correct Answer C. there is a genetic component to each case of Alzheimer's disease. D. cognitive behavioral therapy may extend life expectancy and reverse Alzheimer's symptoms.

B. The passage states that engaging in intellectual pursuits such as chess is correlated with a reduced risk of developing Alzheimer's disease. Because correlation does not imply causation, however, this type of intellectual stimulation may or may not be a causal agent in preventing the disease. Accordingly, it can properly be inferred that engaging in these types of complex tasks may prevent Alzheimer's symptoms (choice B is correct). The passage does not discuss Alzheimer's patients in terms of socioeconomic status, stating only that someone with full-blown Alzheimer's symptoms needs a full-time caregiver (choice A is wrong). The passage states that while there may be a causal link between genetics and Alzheimer's disease, this has only proven to be a factor in a minority of cases (choice C is wrong). Specific therapies such as cognitive behavioral therapy are not discussed in the passage; the symptomatic progression of Alzheimer's disease cannot be reversed at this time through any known means (choice D is wrong).

Unit X is a tight-knit squad of police officers notoriously harsh toward new recruits. One evening, six of the officers devise a plan to "toughen up" two of the newest members; all present agree that the rookies have thus far proven themselves "useless" and "need an attitude adjustment." After responding to a fake emergency call at a remote area, the officers will push the rookies off a well-known low bridge in the area, requiring them to swim a quarter mile to shore. The rookies will then have to find their own way back to the police station. Although a few of the officers silently harbor concerns about the unseasonably cold weather, nobody is willing to openly question the group's plan. Ultimately one of the rookies is overcome by the effects of hypothermia and drowns just prior to reaching shore. Which one of the following factors, if established about Unit X, is likely most responsible for the bad decision to push the new recruits off the bridge? A. The unit polices a high-crime area permeated with hostility towards the police. B. The members of the unit share an extremely strong bond and place prime importance upon loyalty to the group.Correct Answer C. The commanding officers harbor well-known pro-hazing sentiments. D. Nearly all of the officers in the unit come from the same neighborhood, profess similar ideologies, and share very similar ethnic, religious, and socioeconomic backgrounds.

B. The plan to push the rookies off the bridge appears to have resulted from a dysfunctional "groupthink" decision-making process in which members were unwilling to voice dissent. As stated in the passage, the most critical factor leading to groupthink is group cohesiveness (choice B is correct). An external threat, such as a hostile public, is a situational factor that can also lead to groupthink, although it is of less importance than group cohesiveness (choice A is wrong). Similarly, lack of impartial leadership and homogeneity of backgrounds and ideology are considered additional characteristics of a group that are conducive to groupthink as well, although also of less importance than cohesiveness. Furthermore, the fact that members share beliefs and common backgrounds does not imply that the group is necessarily cohesive (choices C and D are wrong).

The theory of social facilitation suggests that participants would be most successful at the marshmallow task if: A. they found the task difficult and were surrounded by peers during the task. B. they found the task easy and were surrounded by peers during the task. Correct Answer C. they were alone in the room during the task. D. they saw their peers succeed at the task.

B. The theory of social facilitation states that individuals perform better at easy tasks when surrounded by others compared to when they perform such tasks in solitude (choice B is correct and choice C is wrong). If the task is difficult, individuals perform worse when surrounded by peers (choice A is wrong). The social facilitation theory does not involve seeing anyone else perform a task, successfully or otherwise (choice D is wrong).

There are five recognized tastes: sweet, salty, bitter, sour, and umami. Recent research suggests there is also a sixth, oleogustus, the unique taste of fat. The lead researcher on the study said that since there aren't any words that exist for this taste, they were forced to make it up, and it was very difficult to figure out if people really view this as unique sensation. This best demonstrates: A. gestalt principles. B. the Sapir-Whorf hypothesis. Correct Answer C. the major limitations of sensory processing. D. monocular cues.

B. This example suggests that it was difficult to describe and define things for which we don't have language; this is an example of the Sapir-Whorf hypothesis, which suggests that the structure of a language determines/influences the thought and behavior characteristic of the culture in which it is spoken. In other words, without an actual word to define the fatty taste, it was difficult to determine that it was in fact a unique taste (choice B is correct). Gestalt psychology emphasizes the mental processes that perceive the "whole" of objects or situations. This school of psychology has been particularly influential in describing tendencies of human visual perception. Gestalt principles do not describe how a lack of language con influence perception (choice A is wrong). This example describes a major limitation of language to define a sensation, not a limitation of sensory processing itself (choice C is wrong). Monocular cues, such as relative size and texture gradient (among others) are cues that can be perceived with only one eye that help us judge distance (depth perception). They have nothing to do with taste (choice D is wrong).

Research indicates that professional burnout can result in thoughts of suicide. Suicidal ideation is most often associated with a psychological disorder in which broad category? A. Anxiety disorders B. Mood disorders Correct Answer C. Personality disorders D. Psychotic disorders

B. Thoughts of death or suicide are most often associated with major depressive disorder, which is a mood disorder characterized by persistent feelings of sadness and hopelessness (choice B is correct). Mood disorders involve disturbances in mood or affect. Anxiety disorders involve excessive worrying and include generalized anxiety disorder, panic disorder, and obsessive-compulsive disorder (choice A is wrong). Research does not show a significant association between these disorders and suicidal ideation, with the exception of post-traumatic stress disorder. Personality disorders involve enduring maladaptive patterns of behavior and cognition and include antisocial personality disorder, avoidant personality disorder, and borderline personality disorder (choice C is wrong). Psychotic disorders involve a loss of contact with reality and include schizophrenia and delusional disorder (choice D is wrong).

Hate crimes committed against immigrants are a consequential example of: A. homophobia. B. xenophobia. Correct Answer C. a social phobia. D. a specific phobia.

B. Xenophobia is defined as the fear of that which is perceived to be foreign, such as the fear of cultural outsiders (choice B is correct). Xenophobic attitudes are one of the negative reactions to immigration, whether involving genuine physical fear or concerns about economic, political, or social factors (such as increased competition for jobs). Homophobia is more complex and is defined as the wide range of negative thoughts and attitudes about non-heterosexual individuals, as well as any resultant behavior (choice A is wrong). A social phobia is defined as the fear of potential embarrassment or humiliation in public situations (choice C is wrong). For example, the fear of using a public restroom is a common example of a social phobia that can influence one's ability to perform routine activities. A specific phobia is defined as the fear of a specific object or situation (choice D is wrong). These are divided into four categories based on the specific triggers: animal, medical, natural environmental, and situational phobias (e.g., arachnophobia is a common specific animal phobia of spiders).

Tony has recently come out to his friends as transgender; despite his female anatomical features, he wants to live and function socially as a man. Tony's friends are very supportive, but it bothers Tony that his friends make such a big deal about his gender transition when introducing him to new acquaintances or when just socializing. Based on this information, which of the following best describes Tony's transgender identity? A. It is his ascribed status. B. It is his achieved status. C. It is his master status. Correct Answer D. It is his status symbol.

C. A master status is one social role (e.g., being transgender) that becomes more prominent than the other social roles that make up one's identity (choice C is correct). Tony has become "the transgender friend". If being transgender were not his master status, Tony might be known as, for example, "the medical student friend who loves to travel and volunteer at the animal shelter and happens to be transgender". An ascribed status is a role that is assigned to someone by society despite that person's efforts to take on another role (choice A is wrong). An achieved status is a role that a person has taken on through his or her own efforts. Although Tony has taken on a role as a transgender person through his own efforts, an achieved status does not become all-encompassing, as a master status does (choice B is wrong). A status symbol is a component of impression management and is generally an object that is displayed in order to effect a certain image. This is unrelated to the information given in the question stem (choice D is wrong).

Which of the following conclusions can be drawn from Figure 1? A. More men reported moderate stress due to lack of time than reported moderate stress due to difficulties with their spouses/partners; fewer women reported that competition with classmates was not very stressful than reported that deadlines were not very stressful. B. Compared to men, a higher percentage of women reported that racism was not very stressful. C. The students who participated in the study were, on average, more stressed about examinations than about competition. Correct Answer D. Financial pressure was the highest source of stress reported by the students who participated in the study.

C. According to Figure 1, higher percentages of male and female students experienced stress due to examinations (Very stressful: 57.4% and 57.9%; Moderately stressful: 30.2% and 28.9%) than experienced stress due to competition with classmates (Very stressful: 25.3% and 29.9%; Moderately stressful: 29.4% and 23.7%) (choice C is the correct answer). While it is true that more men reported moderate stress from lack of time (55.6%) than from difficulties with their spouses/partners (24.8%), more (not fewer) women reported that competition with classmates was not very stressful (46.4%) than reported that deadlines were not very stressful (5.7%; choice A is wrong). Compared to men, a lower (not higher) percentage of women reported that racism was not very stressful (87.6% for women vs. 91.4% for men; choice B is wrong). Figure 1 shows a lower percentage of stress reported for financial pressures than for several other factors such as shortage of time, deadlines, and exams (choice D is wrong).

According to the passage and the data in Figure 1, one way to help anticipatory adults identify more as adults might be to: A. increase the amount of time they have during which they can act independently. B. increase the amount of time they spend with people their own chronological age. C. increase the number of adult responsibilities they have.Correct Answer D. reduce the number of adult responsibilities they have.

C. According to the data in Figure 1, anticipatory adults scored below the mean in terms of feeling that they had enough adult responsibilities. In order to help them identify more with the early adults, who scored high on all four parameters, it might help to increase the number of adult responsibilities they have (choice C is correct). Anticipatory adults did score in the positive range on independence, which was comparable to early adults' scores, so increasing time for independent action would not necessarily help them identify more as adults (choice A is wrong). Increasing the amount of time they spend with people their own chronological age could either help or hinder feelings of adulthood depending on the particular chronological age, and also on the psychosocial maturity levels of the other people (choice B is wrong). Since increasing the number of adult responsibilities would help anticipatory adults identify as adults, reducing the number would have the opposite effect (choice D is wrong).

An individual diagnosed with dissociative identity disorder (DID) is most likely to be comorbid with which one of the following conditions? A. Obsessive-compulsive personality disorder B. Narcissistic personality disorder C. Borderline personality disorder Correct Answer D. Avoidant personality disorder Your Answer

C. Borderline personality disorder involves a pervasive pattern of instability in self-image, mood, interpersonal relationships, and behavior. Comorbidity rates for this condition and DID are quite high. The striking similarities have led to the speculation that DID is actually a manifestation of borderline personality disorder in which the patient erroneously attributes his or her chronic instability to the existence of alternate personalities (choice C is correct). Obsessive-compulsive personality disorder is characterized by rigidity, perfectionism, difficulty with intimacy and handling emotions, and a strong need for control. This profile is very different from the highly unstable one seen in DID (choice A is wrong). Narcissistic personality disorder is characterized by grandiosity, egocentricity, and a compelling need for admiration. DID is not associated with these qualities in particular (choice B is wrong). Avoidant personality disorder is characterized by excessive social inhibition, low self-esteem, and hypersensitivity to rejection and criticism. While some individuals with a DID diagnosis exhibit these traits, they are not prominent features of the phenomenon (choice D is wrong).

Leon Festinger is credited with originating what term to refer to the discomfort and mental tension experienced when two ideas are not congruent, or when a person's behavior does not match his or her attitudes? A. Complementarity B. Reciprocity C. Cognitive dissonance Correct Answer D. Compliance

C. Cognitive dissonance is mental stress or discomfort experienced when an individual's ideas do not comport with his or her behavior, or do not comport with other ideas held by the individual (choice C is correct). The dubious complementarity hypothesis states that an individual will attract those whose needs are different in ways that complement the person's own needs; it does not address internal conflict between behavior and ideas (choice A is wrong). Reciprocity is the social phenomenon in which people's positive deeds put pressure on others to "return the favor", such as with gift-giving. This phenomenon is so powerful that it is said to influence interactions between governments and is one of the motivations for extending international aid (choice B is wrong). Compliance is the act of changing one's behavior in response to the request of another (choice D is wrong).

Suppose that Sarah is kneading a piece of clay in her hands. Which of the following is NOT a strategy relevant to conservation? A. Sarah realizes that the clay is the same piece of clay, regardless of its shape. B. Sarah notices that she can shape the clay into a ball, then a pancake, then back into a ball again. C. Sarah knows that the clay still exists even when her mother puts it back into its container. Correct Answer D. Sarah discovers that squeezing the clay in the middle forces it to become bulkier on the sides.

C. Conservation is the term that Piaget used to describe children's recognition of constancy and consistency in physical matter despite changes in shape or container. This awareness is characteristic of the stage of concrete operational thought, which is usually achieved between the ages of 6 or 7 and 11 or 12. Conservation is realized through three insights: identity, reversibility, and reciprocity. Identity involves the child's recognition that the physical object is the same object, regardless of how it is manipulated (choice A is a strategy relevant to conservation and is therefore wrong). Reversibility involves the child's recognition that a given manipulation can be reversed to give the object its initial appearance (choice B is a strategy relevant to conservation and is therefore wrong). Finally, reciprocity involves the child's realization that a manipulation of one dimension or aspect yields a corresponding change in another aspect. As a result, the manipulation is understood to change the original object rather than create a new one (choice D is a strategy relevant to conservation and is therefore wrong). The realization that an object continues to exist despite its disappearance from the visual field is known as object permanence (choice C is not a strategy relevant to conservation and is therefore correct).

Optimism, a term which describes the tendency to expect the best outcomes, is a protective factor against which of the following disorders? A. Attention Deficit Hyperactivity Disorder B. Autism C. Depression Correct Answer D. Learning Disability

C. Depression is characterized by pervasive and persistent low mood accompanied by low self-esteem and other vegetative (physical) symptoms, such as decreased or increased desire for sleep. Research shows that optimistic people have a much lower rate of developing depression in their lifetimes, which may be partly attributable to their positive self-talk. Increasing positive self-talk is a known Cognitive Behavioral Therapy technique for treating depression (choice C is correct). ADHD is a physiological condition resulting from neurological conditions, and is not connected to the individual's outlook on life, though his/her ability to cope with such a disorder is likely directly impacted by the level of optimism/pessimism that is characteristic (choice A is wrong). Autism is a biologically-based neurodevelopmental disorder, the development of which is not influenced by the temperament of the individual (choice B is wrong). The development of learning disabilities, similar to that of ADHD, is not directly influenced by the degree of optimism that a child develops, though the ability to cope and manage such a disorder is likely influenced by the individual's optimism (choice D is wrong).

Suppose that one study found that, during hypnosis, electroencephalograph (EEG) measurements of the brain demonstrate predominantly high amplitude, low frequency delta waves. If true, brain activity during hypnosis is most similar to what sleep stage? A. Stage 1 B. Stage 2 C. Stage 3 Correct Answer D. REM

C. EEG recordings during Stage 3 sleep demonstrate delta waves, particularly in later Stage 3 sleep during which these waves predominate (choice C is correct). EEG recordings during Stages 1 and 2 demonstrate a predominance of lower amplitude, higher frequency waves called theta waves (choice A is wrong); while K-complexes, which are lower frequency, higher amplitude waves, do occur during Stage 2 sleep, these waveforms do not predominate during this stage (choice B is wrong). EEG recordings during REM sleep, also known as paradoxical sleep, demonstrate lower intensity waves with variable frequency that most closely resemble the beta waves recorded during wakefulness (choice D is wrong).

Though the bond between parent and child begins at birth, attachment patterns can be better identified once the child becomes mobile and is able to explore his or her surroundings. Thus, birth to 18 months is a critical time in which attachment bonds form and in which attachment styles begin to be observed and solidified. Erik Erikson's first stage of development, which also encompasses birth to 18 months, coincides nicely with attachment theory and its tenets. What is the basic conflict in Erikson's first stage? A. Initiative vs. Guilt B. Industry vs. Inferiority C. Trust vs. Mistrust Correct Answer D. Autonomy vs. Shame and Doubt

C. Erik Erikson described eight stages of psychosocial development that occur throughout the lifespan. Each stage reflects a struggle between two tendencies; resolution of each conflict results in a healthy personality and the acquisition of basic virtues. Erikson's first stage occurs during infancy (birth to 18 months), and its basic conflict is trust vs. mistrust. At this stage, the child develops a sense of trust when caregivers provide consistent and reliable care and affection. This is akin to the formation of a secure attachment as described by attachment theory (choice C is correct). The third stage in Erikson's model occurs during the preschool years (3 to 5 yrs) and the conflict to be resolved is initiative vs. guilt. Children begin asserting control and power over the environment, and success at this stage leads to a sense of purpose. However, if the child experiences disapproval as a result of his or her attempts to assert control and power, a sense of guilt is instilled (choice A is wrong). The fourth stage in Erikson's model occurs during the school-age years (6 to 11 yrs) and deals with the conflict of industry vs. inferiority. Children are in the midst of coping with new academic and social demands and success leads to a sense of competence, while failure results in feelings of inferiority (choice B is wrong). The second stage in Erikson's model occurs during early childhood (18 months to 3 yrs). During this stage, the child must resolve the conflict of autonomy vs. shame and doubt. Children begin to assert their independence by discovering and using different skills and abilities, such as putting clothes and shoes on and becoming toilet-trained. Success leads to feelings of autonomy, while failure results in feelings of shame and doubt (choice D is wrong).

"M" is in police custody undergoing the ninth straight hour of interrogation regarding the death of his wife, whom he discovered shot to death that evening upon returning home from work. For the past hour detectives have been continuously yelling at him in an effort to get him to admit responsibility for the crime. Unable to endure the situation any longer, M states that he must have shot his wife but simply can't remember. In a soft and comforting tone the detectives praise M for making the admission, then gently ask him to try to remember the details of the crime. When he says that he cannot, the detectives resume yelling at him. M then capitulates, stating that he remembers having shot his wife. The detectives then gently ask M a lengthy series of questions regarding the supposed incident, in response to which M offers vivid and detailed "recollections" of the shooting. M's recounting of these purported details of killing his wife is an example of: A. escape learning. B. positive punishment. C. avoidance learning. Correct Answer D. positive reinforcemen

C. Escape learning occurs when an individual learns a means of escaping an unpleasant stimulus. M's first statement indicating that he must have shot his wife but can't remember doing so, and his second statement indicating that he in fact remembered the incident, are examples of escape learning. However, escape learning turned into avoidance learning when M became able to prevent the aversive stimulus (the yelling) from occurring in the first place by engaging in the behavior (i.e., making inculpatory statements; choice C is correct and choice A is wrong). Positive punishment involves decreasing a behavior by following it with an undesirable or unpleasant stimulus. M's behavior (making inculpatory statements) was increased, not decreased (choice B is wrong). Positive reinforcement involves increasing a behavior by following it with a desirable or pleasurable stimulus. Yelling is an unpleasant stimulus (choice D is wrong).

If children in a low socioeconomic status (SES) group were shown to have a significantly shorter mean wait time than children in a high SES group, what might that suggest, based on the information provided in the passage? A. Low SES children eat smaller meals on average than do high SES children. B. Low SES children eat larger meals on average than do high SES children. C. Low SES children may be told that they will have three meals each day, but often may not. Correct Answer D. Low SES children may often be accurately told that they will not have three meals each day.

C. Figure 1 indicates that children in the unreliable condition (i.e., the one wherein experimenters do not fulfill their promises) show a significantly shorter mean wait time. Therefore, the conclusion that can be drawn from the passage is that if a child's home life is similarly unreliable (a possibility for those in the low SES group), he or she will likely demonstrate a short mean wait time (choice C correct). An inference about the size of the children's meals is not supported by information in Figure 1 (choices A and B are wrong). If children are accurately told that they will not have three meals each day, their respective environments may not be healthful but they will not be "unreliable" as demonstrated in the study. Therefore, it cannot be assumed that these children will have a lower mean wait time based on the passage (choice D is wrong).

Which of the following can be concluded about the results of the study described above? A. Anxiety is only dependent upon level of worry, not upon rumination. B. Those with depression are more likely to experience anxiety with worry compared to those with GAD who worry. C. Worry and rumination both play a role in creating one's level of anxiety. Correct Answer D. Those with depression are less likely to experience anxiety with rumination compared to those with GAD who ruminate.

C. For individuals with GAD and also for those with depression, the rumination and worry inductions produced significantly elevated levels of anxiety compared to the control condition. Therefore, it can be concluded that both worry and rumination play a role in anxiety (choice C is correct). Further, based on this information, it would be incorrect to state that anxiety is only dependent upon level of worry, and not upon rumination (choice A is wrong). Figure 1 shows those with GAD experiencing a slightly higher level of anxiety in the worry condition than those with depression in the worry condition. Accordingly, it is incorrect to state that those with depression are more likely to experience anxiety with worry compared to those with GAD who worry (choice B is wrong). Furthermore, Figure 1 shows those with depression experiencing a slightly higher level of anxiety in the rumination-induction condition than those with GAD in the rumination-induction condition. Therefore, it is also incorrect to state that those with depression are less likely to experience anxiety with rumination compared to those with GAD who ruminate (choice D is wrong).

According to Figure 1, which of the following indicators had the smallest socioeconomic gap between blacks and whites? A. Median household income B. Median net worth C. Median net worth, lowest income quintile Correct Answer D. Median net worth, highest income quintile

C. In Figure 1, the "median net worth, lowest income quintile" indicator had the smallest socioeconomic gap between the blacks and the whites, with a difference of roughly one thousand dollars (choice C is correct). Compared to the "lowest income quintile" indicator, the "median household income" and "median net worth" indicators both had a significantly higher socioeconomic gap between the blacks and the whites, with a difference of over ten thousand dollars (choices A and B are wrong). The "highest income quintile" indicator had the highest socioeconomic gap of all the indicators, with a difference of nearly one hundred thousand dollars (choice D is wrong).

NPM-2005 did not account for the personal experiences of the reported families. This is a suggestion for future directions in research. Researchers predict that the aversive impact of stressful life events might explain the elevated rates of some forms of child maltreatment in certain groups. This effect is expected to be the most pronounced in: A. native families. B. traditional immigrant families. C. non-traditional immigrant families. Correct Answer D. all immigrant families.

C. It is true that all families, both native and immigrant families, experience some stressful life events. However, this stress is expected to be highest in non-traditional immigrant families, which are defined as refugees (choice C is correct). It is not expected that native families will tend to be impacted by stressful life events at a greater rate than immigrant families (choice A is wrong). In contrast to traditional immigrant families, which tend to emigrate from former Dutch colonies, non-traditional immigrant families are more heterogeneous in terms of their ethnicities. However, the fact that these groups migrated due to persecution suggests a shared experience that might contribute to traumatization (choices B and D are wrong). Furthermore, the complicated process of gaining political asylum also suggests more post-migration stress. This is significant when considering rates of child maltreatment. For example, this stress could lead to the development of parental disorders, like post-traumatic stress disorder (PTSD). This involves three clusters of symptoms, including physiological hyperarousal, which might include angry outbursts.

Which of the following is/are true regarding Gestalt laws of perceptual organization? I. Certain basic features of vision, including color and motion, are processed consciously in a bottom-up manner II. In our visual field, we tend to group nearby objects together; this is known as "proximity" III. When viewing something, we tend to fill in gaps in order to complete whole objects; this is known as "closure" A. II only B. III only C. II and III only Correct Answer D. I, II, and III

C. Item I is false: basic visual features, such as color and motion, are processed instantly and automatically by the brain, not consciously in a bottom-up fashion (choice D can be eliminated). Item II is true: one of the Gestalt principles of grouping is that of proximity; we tend to group nearby figures or objects together (choice B can be eliminated). Item III is true: another of the Gestalt principles of grouping is that of closure; we tend to fill in gaps in order to perceive objects that are whole and complete (choice A can be eliminated and choice C is correct).

There are multiple sociological perspectives on deviance. Which of the theorists below are expected to consider the associated questions about non-normative behaviors? I. Differential association theorists; "How can people resist deviance?" II. Labeling theorists; "Who defines deviance?" III. Structural strain theorists; "How do norms affect deviance?" A. III only B. I and II only C. II and III only Correct Answer D. I, II, and III

C. Item I is false: differential association theorists argue that deviant behaviors are learned through interactions between individuals and their communities. These theorists contend that people become deviant when they are exposed to deviant behavior and attitudes. "How can people resist deviance?" is therefore not a pertinent question (choices B and D can be eliminated). The main criticism of differential association is that it essentially reduces individuals to their environments. Because it states that those in deviant communities learn to be deviant themselves, the possibility of resistance is not contemplated. Item II is true: labeling theory asserts that behaviors are seen as deviant as the result of social processes of labeling. "Who defines deviance" is therefore a pertinent question (choice A can be eliminated). Labeling theorists do indeed address this concern; for example, these theorists are interested in the mechanisms through which power contributes to deviance labeling (e.g., agents of social control). Item III is true: structural strain theory suggests that deviant behaviors are the result of tension between the accepted social goals and the institutionalized means available to achieve those goals. "How do norms affect deviance?" is therefore a pertinent question (choice C is correct). Structural strain theorists consider the effects that social norms have on behavior, normative or not, and suggest that there is pressure to use deviant methods when the social structure does not support the achievement of the accepted goals.

Suppose that a young unwed teenager who dropped out of high school and frequently uses drugs gets pregnant and has a daughter. In high school this daughter begins to associate with a crowd that uses drugs, experiments with drugs herself, has unprotected sex, and is on the verge of getting expelled from school. Which of the following best explains what might be occurring in this scenario? I. Downward mobility II. Intergenerational mobility III. Social reproduction A. I only B. II only C. III only Correct Answer D. II and III only

C. Item I is false: downward mobility describes a lowering in social class for an individual. There is not enough information to ascertain that the social class of either the mother or the daughter has declined. In fact, since the daughter is behaving in ways similar to how her mother behaved at her age, it might be inferred that the social class of both individuals has remained the same (choice A can be eliminated). Item II is false: intergenerational mobility occurs when there is a change in social class between parents and children within a family; again, there is not enough information to ascertain if this is the case, and, if anything, it appears as though the mother and daughter are similar in terms of social class (choices B and D can be eliminated). Item III is true: social reproduction occurs when structures and activities in place within a society serve to transmit and reinforce social inequality from one generation to the next. Cultural and social capital are two mechanisms by which social reproduction occurs. Cultural capital (such as education) affords one the potential to be upwardly mobile, as does social capital (one's social network). Since the daughter is not utilizing either cultural capital or social capital to her advantage, she seems to be making some of the same choices that her mother made. Therefore, social reproduction best explains this scenario (choice C is correct).

Which of the following is/are supported by the passage? I. Limited ability to delay gratification may be a factor in adult obesity II. Stability in a child's environment could impact his later academic success III. Genetics do not appear to play a role in children's ability to delay gratification A. I only Your Answer B. II only C. I and II only Correct Answer D. I, II, and III

C. Item I is true: the first paragraph mentions that children's ability to delay gratification predicted BMI scores later in life, implying that children who performed worse on the delayed gratification task might show BMIs in the "obese" range as adults (choice B can be eliminated). Item II is true: the first paragraph also mentions that success at the marshmallow task predicted greater academic success later in life, and, when combined with the results of the study described in the second paragraph, implies that a stable environment could lead to greater academic success (choice A can be eliminated). Item III is false: the study shows that environment plays a role in one's ability to delay gratification, but the passage makes no mention of genetics (choice D can be eliminated and choice C is correct).

One advantage of using a longitudinal study design, such as the one described in the passage, is: A. the ability to conduct in-depth analyses of a few participants to get very rich and detailed data samples. B. the opportunity to follow a cross-section of participants of different types at the same time. C. the opportunity to follow one group of participants over an extended period and track their changes. Correct Answer D. the ability to see how many participants remain in the study and how many people withdraw.

C. Longitudinal studies in developmental psychology research, such as the one in the passage, allow researchers to follow one group of participants over time. In this study, they were able to follow participants over a six-year period and track their changes (choice C is correct). Case study design (not a longitudinal study design) allows researchers the opportunity to conduct in-depth analyses of a few select participants, then emerge with very detailed data (choice A is wrong). Cross-sectional research (not a longitudinal study design) gives researchers a variety of participants of different types (e.g., different ages) that they can track at the same time (choice B is wrong). A drawback (not an advantage) of longitudinal research is that sometimes participants will withdraw from the research over the period of the study for a variety of reasons, including moving away, lack of interest, or even illness or death (choice D is wrong).

Managing stigma can be very stressful. Which of the following hormones is NOT related to the stress response? A. Cortisol B. Epinephrine C. Oxytocin Correct Answer D. Norepinephrine

C. Oxytocin is a well-known hormone associated with bonding, particularly between lovers or between parents and their children; oxytocin is not involved in the stress response (choice C is correct). Epinephrine, cortisol, and norepinephrine are all associated with the endocrine stress response. Epinephrine and norepinephrine are associated with the "fight or flight" response to stress and increase the body's ability to move vigorously (choices B and D are wrong). Cortisol is a glucocorticoid that can increase blood sugar and aid in metabolism, which aids the stress response by ensuring that the body has the required immediate energy stores to mobilize a response to the stress trigger (choice A is wrong).

Two brothers, James and Paul, are eight and six years old, respectively. Lately, James has seemed much more advanced than his younger brother, which their mother attributes to birth order. She assumes that James works harder because he is the first born, while Paul seems to think that "the world revolves around him". However, when she takes the children to a psychologist, he suggests that their behaviors are instead reflective of Piaget's stages of cognitive development. Which of the following is true of the boys in terms of Piaget's stages? A. James is in the formal operational stage, and Paul is in the preoperational stage. B. James is in the formal operational stage, and Paul is in the sensorimotor stage. C. James is in the concrete operational stage, and Paul is in the preoperational stage. Correct Answer D. James is in the concrete operational stage, and Paul is in the sensorimotor stage.

C. Piaget described four stages of cognitive development: the sensorimotor stage, the preoperational stage, the concrete operational stage, and the formal operational stage. The sensorimotor stage generally occurs when children are between birth and two years old (choices B and D can be eliminated). The preoperational stage generally occurs when children are between two and seven years old. At this stage, children tend to be egocentric, viewing the world only from one perspective. Based on the question stem, it would seem that Paul is in the preoperational stage. Next, the concrete operational stage occurs when children are about seven to eleven years old. Finally, the formal operational stage usually occurs when children are aged twelve or older (choice A can be eliminated). Since James is eight, it would appear that he is in the concrete operational stage (choice C is correct).

Efforts to teach algebra to upper-level elementary students are often more successful when they involve concrete representations of algebraic processes, such as using blocks on a balance scale, than when they simply utilize symbolic processes (letters and numbers). One explanation for this is presented in: A. Vygotsky's zone of proximal development. B. Erikson's psychosocial theory. C. Piaget's stages of development. Correct Answer D. Freud's psychosexual theory.

C. Piaget's theory suggests that older elementary students are still in the concrete operational stage and are accordingly unable to perform abstractions, such as those required for algebra, until they enter the formal operational stage; Piaget's theory best explains the scenario presented (choice C is correct). Vygotsky's zone of proximal development refers to the gap between what a student has already mastered and what he or she could master if provided with educational support. This is not directly related to age and no mention is made of students' previous knowledge (choice A is wrong). Erikson's psychosocial theory has no direct application to learning algebra (choice B is wrong), and Freud's theory does not deal with educational readiness (choice D is wrong).

In the study presented in the passage most physicians reported: A. high emotional exhaustion, low depersonalization, and low personal accomplishment. B. low emotional exhaustion, high depersonalization, and low personal accomplishment. C. low emotional exhaustion, low depersonalization, and high personal accomplishment. Correct Answer D. low emotional exhaustion, low depersonalization, and low personal accomplishment.

C. Reports of the three measures of professional burnout are presented in Figure 2. The first chart presents reports of depersonalization; the most common response is low depersonalization, accounting for 50% of respondents (choice B can be eliminated). The second chart presents reports of emotional exhaustion; the most common response is low emotional exhaustion, accounting for 42% of respondents (choice A can be eliminated). The third chart presents reports of personal accomplishment; the most common response is high personal accomplishment, accounting for 66% of respondents (choice D can be eliminated and choice C is the correct answer).

It can be concluded that the physician and non-physician samples reported the greatest professional differences in which area? A. Professional burnout; item = "emotional exhaustion" B. Professional burnout; item = "depersonalization" C. Professional experience; item = "average hours worked per week" Correct Answer D. Professional experience; item = "work schedule permits sufficient time for personal life"

C. Result comparisons are presented in Figure 1, which includes two measures of professional experience and two measures of professional burnout. Based on the results of these four measures, the greatest differences in the samples are those in professional experience, and the greatest difference overall concerns the average number of hours worked per week (choice C is correct). The results show that 39% of physicians and 12% of non-physician respondents reported working 60 hours or more per week; thus, this schedule range is more than three times as common among physicians as non-physicians. The other measures show less drastic differences between the sample groups (choices A, B, and D are wrong).

A secure attachment style is best demonstrated by which of the following scenarios? A. A five-year-old child who remains by her mother's side at a children's birthday party, not interacting with the other children B. A four-year-old child who runs toward her mother with open arms upon her return from work, but before reaching her turns around and instead runs away frightened. C. A four-year-old who confidently joins in a wide array of activities while at a birthday party and later seeks his mother's help, and is soothed by her, when a game of tag results in a skinned knee Correct Answer D. A five-year-old child who explores different parts of a playground, pushing his climbing of the jungle gym to dangerous heights, with the knowledge that his mother will not be paying attention

C. Secure attachment is promoted by a responsive and available parent and is characterized by a child who is able to explore his or her environment secure in the knowledge that the parent will be there when needed. A child confidently exploring and joining in different activities at a birthday party demonstrates a secure attachment style. The child's act of seeking his mother's aid during a stressful time, such as when being injured, and the parent being available to comfort and soothe him, further displays secure attachment (choice C is correct). Reluctance to explore and instead clinging and remaining by the mother's side is indicative of an anxious-resistant insecure attachment style, in which the child is uncertain about the parent's availability because of previous separations. Because of this fear of being abandoned, he or she stays close and clings to the mother (choice A is wrong). Odd or awkward behaviors when separating or reuniting with the parent, such as extreme shifts between proximity-seeking and avoidance, are indicative of a disorganized attachment style. A child who, upon being reunited with her mother, initially runs toward her with open arms and then quickly runs away instead illustrates an extreme shift between proximity-seeking and avoidance. Disorganized attachment is usually the result of the child experiencing trauma, such as being physically, emotionally, or sexually abused by the parent. In this instance, the child sees the parent as both frightening and the only source of comfort, which causes great confusion and results in the eratic proximity-seeking and avoidance behavior (choice B is wrong). An anxious-avoidant attachment style results from the child being constantly rejected and rebuffed by the parent. These children learn to be independent early on, depending only on themselves because they learn that they cannot rely on the parent who is never available. A child exploring and climbing the jungle gym to dangerous heights displays this early independence. The child's knowledge about the lack of availability and responsiveness of the parent further provides evidence of an anxious-avoidant attachment style (choice D is wrong).

Larry is a hardworking student who always earns high grades. When he is assigned to work on a group project with four of his classmates, however, his motivation seems to dwindle. Instead of pressing on for hours as he normally would for an individual assignment, he develops a lackadaisical attitude toward the project. Which of the following can most accurately explain his change in attitude? A. Social facilitation B. Assimilation C. Social loafing Correct Answer D. Peer pressure

C. Social loafing occurs when individuals exert less effort than they normally would because they are working in a group setting. In this scenario, submaximal goal-setting is apparent in that Larry does not expect that he will need to do as much work since others will be working toward the final outcome as well (choice C is correct). Like social loafing, social facilitation takes place in group settings. However, social facilitation occurs when individuals perform simple or well-practiced tasks better when they know others are watching (choice A is wrong). In sociology, assimilation is the process by which individuals, often immigrants, become absorbed into their new society (choice B is wrong). Peer pressure occurs when an individual feels pressured by his or her peers to act a certain way; there is no indication in the question stem that Larry's group members suggested that he put in less effort (choice D is wrong).

When an individual joins a social group, the process by which that individual learns the norms, values, and customs of that social group is called: A. formal operations. B. assimilation. C. socialization. Correct Answer D. membership.

C. Socialization is defined as the process of learning the norms, values, and customs of a new social group (choice C is correct). Formal operations is a stage in Piaget's theory of cognitive development wherein children can engage in abstract reasoning (choice A is wrong). Assimilation describes the process by which one social group becomes incorporated into another, generally in the context of immigration, by taking up the traditions of the new group and leaving behind their old traditions. Assimilation is a group-level theory, while socialization is an individual-level theory (choice B is wrong). Membership marks the status of the individual after he or she has learned the norms, values, and customs of the new group, not the process by which the individual learns them (choice D is wrong).

What is the main premise of the behaviorist theory of personality? A. Personality is the result of a person's multiple traits. B. Personality is the result of a person's unconscious mind and childhood experiences. C. Personality is the result of interactions between the person and the environment. Correct Answer D. Personality is the result of a person's genetics.

C. The behaviorist theory of personality emphasizes that personality is derived from the interactions between a person and his or her environment (choice C is correct). Trait theory, a type of humanist theory, emphasizes that personality is the result of a person's multiple traits that are relatively stable over time (choice A is wrong). Psychodynamic theories emphasize that personality is the result of a person's unconscious mind and childhood experiences (choice B is wrong). Biological theories of personality emphasize that personality is the result of a person's genetics (choice D is wrong).

Conflict theorists are concerned with stratification in societies. Which of the following arguments is LEAST consistent with this perspective? A. Stratification is the result of competition between different social classes. B. Stratification is the result of the exploitation of subordinate groups. C. Stratification is the result of the need for maintenance of social order. Correct Answer D. Stratification is the result of capitalist motivation to accumulate wealth.

C. The conflict perspective asserts that social structures, including structured social stratification, reflect the competition for limited resources (choice A is consistent with conflict theory and can be eliminated). These structural inequalities exist because those with the most power (the dominant groups) maintain their positions of power through the suppression of those with the least power (the subordinate groups; choice B is consistent with conflict theory and can be eliminated). The traditional conflict theorists found this idea to be of particular interest with respect to capitalist societies. Marx, for example, argued that capitalism produces internal tensions between those who control the means of production and those who labor for the production (choice D is consistent with conflict theory and can be eliminated). However, the argument that stratification contributes to social order would not likely be made from the conflict perspective; this is better described as a functionalist argument (choice C is not consistent with conflict theory and is the correct answer). For example, Marx argued that the tensions associated with capitalism would lead to its self-destruction, suggesting that he did not find pronounced stratification to contribute to social order.

Based on information provided in the passage, most religious women who have had an elective abortion are reported members of which of the following? A. An ecclesia B. A cult C. A church Correct Answer D. A sect Your Answer

C. The first paragraph presents the most common demographic characteristics of those who reported having had an abortion, including the fact that most of the women are religious with the most represented group being Protestants. There are four main forms of religious organizations (from the most to the least widespread): ecclesia, churches, sects, and cults or new religious movements. Catholicism and Protestantism form the basis for two of the most common churches in the United States (choice C is correct). These religious organizations are integrated into the larger societies but continue to maintain specific rules and regulations specific to the religion. The Protestant church in the United States is considered a denomination as it is independent of the state. Ecclesia are dominant religious organizations that are recognized as the national religion; for example, Islam is Iran's state religion. There is no official ecclesia in the United States due to the separation of the church and state (choice A is wrong). Cults are religious organizations that are separate from their large societies, and their beliefs and practices are thought to be far outside of the accepted social norms, such as the Branch Davidians known for the Waco siege (choice B is wrong). Sects are religious organizations that are separate from their larger societies, often as a result of separation from a larger church, such as the Amish and Mormon communities (choice D is wrong).

Jackson is nearly involved in a car accident; when starting out at an intersection, he witnesses another driver running a red light. According to the fundamental attribution error, he is most likely to believe that the other driver: A. must have had a good reason for running the red light. B. is running away from the police. C. is careless and/or reckless. Correct Answer D. is having mechanical problems with his breaks.

C. The fundamental attribution error is the tendency of individuals to place blame for another person's behavior on internal factors (in contrast to the tendency to blame external events when explaining their own behaviors). Thus, the driver who witnesses another driver run a red light is likely to assume that this behavior speaks to the character of the other driver, rather than to external circumstances (choice C is correct). Blaming external events is more likely when a person explains his or her own behavior; when explaining others' behavior, individuals tend to attribute the behavior to internal characteristics (choice A is wrong). There is no reason to assume criminality on the part of the other driver. Further, doing so would involve an external explanation for the behavior, which is less likely according to the fundamental attribution error (choice B is wrong). Mechanical problems would also be an external explanation for the behavior of the other driver, and would therefore be less likely according to the fundamental attribution error (choice D is wrong).

Based on the passage, ADHD can best be described as: A. a disorder of the limbic system that is mediated through neurological and behavioral pathways. B. a mood disorder that impacts academic functioning. C. a frontal lobe disorder that can be partially managed through behavioral techniques. Correct Answer D. a severe anxiety disorder that responds to medication.

C. The passage emphasizes that the core symptom categories of ADHD, inattentiveness and impulsivity, derive from executive dysfunction. Executive functioning, such as inhibition and working memory, is associated with the frontal lobe, and choice C is the only choice that implicates the frontal lobe in the disorder (choice C is correct). While ADHD seems to have both behavioral and neurological bases, executive functions are not associated with the limbic system (choice A is wrong). Further, although ADHD does impact academic performance and is often stabilized with medication, the passage describes ADHD as a disorder of executive functioning and not as a mood or anxiety disorder (choices B and D are wrong).

According to the passage, symptoms of depression include all of the following EXCEPT: A. anhedonia. B. blood flow irregularities in the frontal cortex. C. an alternating pattern of mood swings. Correct Answer D. high amounts of circulating cortisol.

C. The passage never mentions mood swings as a symptom of depression (choice C is not a symptom of depression and is therefore the correct answer). Anhedonia is the inability to feel pleasure. In the last paragraph the passage states that depressed patients "prominently exhibit a reduced ability to experience pleasure" (choice A is true and can be eliminated). In the second paragraph, the passage mentions that imaging studies have revealed blood flow irregularities in the hippocampi and the frontal cortexes of depressed patients (choice B is true and can be eliminated). The passage also states that decline in hippocampal function can lead to hypercortisolemia or elevated levels of circulating cortisol (choice D is true and can be eliminated).

The reported findings for Miami, Florida might be explained by: A. decreases in the racist policies that contribute to racial segregation. B. increases in the economic progress of minorities in the state of Florida. C. increases in the rate of immigration to the United States.Correct Answer D. increases in the efforts to decrease residential segregation in central cities.

C. The passage states that Miami, Florida is one of the cities that experienced increases in residential segregation, as assessed using the data from the past two census reports. Therefore, the correct answer should reflect a change that is expected to cause increases in racial disparities. The rate of immigration to the United States has had profound effects on the demographic composition of the nation's largest metropolitan areas. Recent immigrants to the United States are most often Hispanic, and this is heightened in Miami, which has a Hispanic population share of 66% based on the most recent census data. It is also known that immigrants tend to be located in ethnic communities ("ethnic enclaves"), which suggests residential segregation (choice C is correct). The decrease in racist policies responsible for the historic concentration of minorities in poor neighborhoods explains much of the decline in residential segregation nationwide; it does not explain the observed increase in segregation in Miami (choice A is wrong). The increase in formal policies prohibiting racism in the United States, one of the modern social changes of importance for the MCAT, has contributed to the increased economic progress (not regression) of minorities (choice B is wrong). It has been noted, for example, that the rise of the black middle class has contributed to the decline in residential segregation; this can be attributed, in part, to the migration of these black Americans to the suburbs. The increase in efforts to decrease residential segregation would suggest a decline in such segregation; it does not explain the observed increase in racial segregation in Miami (choice D is wrong).

Access to, and utilization of, preventative care is often evaluated through a community's use of prophylactics, defined as medical or public health procedures that prevent disease. Immunization is a common and widespread example of prophylaxis. Additional examples of prophylaxis would be expected to include all of the following EXCEPT: A. birth control methods, including condoms. B. regular moderate physical activity. Your Answer C. cesarean section (c-section) delivery. Correct Answer D. fluoride therapy and tooth cleaning.

C. The question describes prophylaxis as preventative care. Birth control can be used as a preventative measure with respect to pregnancy (choice A is an example of prophylaxis and can be eliminated). Furthermore, condoms can also be used to prevent sexually transmitted disease. Physical activity has numerous health benefits, including the ability to reduce one's risk of developing conditions such as heart disease, muscular and skeletal diseases, and even some forms of cancer (choice B is an example of prophylaxis and can be eliminated). Fluoride therapy and tooth cleaning help to prevent dental diseases, such as gingivitis, which can have a negative effect on overall health (choice D is an example of prophylaxis and can be eliminated). Cesarean section, although a common procedure, is a surgical method. It is performed in response to complications when physicians deem it a safer method of delivery; it is generally not used preventatively (choice C is not an example of prophylaxis and is the correct answer).

The results of the experiment described in the passage suggest which of the following about the person-situation debate? A. Behavior is determined solely by traits. B. Behavior is determined solely by the situation. Your Answer C. Behavior is determined by a combination of traits and the situation. Correct Answer D. Behavior is determined neither by traits nor by the situation.

C. The researchers in the above study found that children's level of hostility varied across situations. However, children also had highly stable situation-behavior profiles. The variability of behavior suggests that situational factors influence behavior, while the stable situation-behavior profiles suggest that traits also influence behavior (choice C is correct). The variability in behavior across situations shows that behavior is not determined solely by stable traits (choice A is wrong). The consistency of the situation-behavior profiles shows that the situation alone does not determine behavior (choice B is wrong). The results of the experiment do not support the idea that neither traits nor situational factors influence behavior (choice D is wrong).

Non-traditional immigrant families are overrepresented in reports of all of the following, EXCEPT: A. emotional abuse. B. physical abuse. C. sexual abuse. Correct Answer D. other abuse.

C. The results are presented in Figure 1. It is important to note that non-traditional immigrants accounted for 12 percent of the participant sample (paragraph 4). Reports of child maltreatment are higher than expected based on this representation in the sample in the following categories: emotional abuse (25 percent), physical abuse (17), and other abuse (16; choices A, B, and D can be eliminated because non-traditional immigrant families are overrepresented in these reports). However, non-traditional immigrants account for a mere six percent of reports of sexual abuse, which is lower than expected based on their representation in the sample (choice C is correct as non-traditional immigrant families are underrepresented here).

In the figures below, if the solid black lines represent those individuals who scored high on the "Entity" view of intelligence, and the dashed gray lines represent those individuals who scored high on the "Incremental" view of intelligence, which of the following supports the researchers' hypothesis?

C. The second paragraph states that the researchers hypothesized that entity theorists facing a difficult comprehension task would perceive their learning to be poorer than those facing an easier comprehension task. In contrast, incremental theorists' perception of learning would not change, regardless of the level of comprehension difficulty. The third paragraph further explains that the high coherence text was easier to comprehend than the low coherence text. Accordingly, the researchers would expect that entity theorists' perceived comprehension would be higher in the high coherence condition than in the low coherence condition, while the incremental theorists' perceived comprehension would remain stable in both conditions. Since the black lines represent those individuals who scored high on the "Entity" view of intelligence, and the dashed gray lines represent those individuals who scored high on the "Incremental" view of intelligence, the figure in answer choice C shows the entity theorists' perceived comprehension decreasing from the high coherence condition to the low coherence condition, while the incremental theorists' perceived comprehension remains stable; this is consistent with the hypothesis in the second paragraph (choice C is correct). The figure in choice A shows the opposite effect, as entity theorists' perceived comprehension remains stable and the incremental theorists' perceived comprehension decreases from the high coherence condition to the low coherence condition (choice A is wrong). The figure in choice B is correct regarding the expected trend for entity theorists, but it also shows incremental theorists' perceived comprehension increasing in the high coherence condition. Such an increase is not supported by the information in the passage (choice B is wrong). The figure in choice D is correct in terms of the expected trend for incremental theorists, but it also shows an increase in perceived comprehension among entity theorists, which contradicts the researchers' hypothesis (choice D is wrong).

"T" has been wetting the bed consistently for the past five months. His parents decide to implement a "star chart" system to help him track his wet and dry nights. T gets a gold star on the chart for every morning that he wakes up dry. This conditioning strategy is an example of: A. extinction. B. positive punishment. C. positive reinforcement. Correct Answer D. negative reinforcement.

C. This question draws on the principles of operant conditioning. In the case of T, his parents want him to decrease a certain behavior (bedwetting). When he does not wet the bed, he receives a gold star for his star chart. This is an example of positive reinforcement; when the desired behavior (waking dry) occurs, it is rewarded. In accordance with operant conditioning principles, this should help T to stop wetting the bed (choice C is correct). In operant conditioning, extinction is the process whereby a previously reinforced behavior is no longer reinforced. After the reinforcer has been eliminated, the behavior will eventually be extinguished. T's parents are reinforcing a behavior (waking dry), not trying to extinguish a previously reinforced behavior (choice A is wrong). Positive punishment occurs when a behavior is met with an aversive stimulus designed to reduce the frequency of the behavior. If T's parents spanked him every time that he wet the bed, this would be an example of positive punishment (choice B is wrong). Negative reinforcement occurs when a desired behavior is reinforced by removing an aversive stimulus. In order to avoid the aversive stimulus, the person or animal being trained will increase the behavior (choice D is wrong).

Social stratification can create multiple hierarchical threats for certain individuals. The concept of triple jeopardy describes the specific situation in which a person's different social statuses interact with one another so that the negative effects of each social status contribute to negative social experiences and to a lower quality of life. This population is expected to include: A. black men with an average age of 25. B. white men with an average age of 75. C. black women with an average age of 75. Correct Answer D. white women with an average age of 25.

C. This question requires an understanding of social trends of discrimination as a result of age, race, and sex in order to determine which individuals are expected to have the lowest statuses. Modern societies often discriminate against older individuals, black individuals, and women. These ageist, racist, and sexist attitudes are expected to pose the greatest threat ("triple jeopardy") to older black women (choice C is correct). The average black man who is 25 may experience a single form of discrimination (racism) and can still benefit from his other statuses (choice A is wrong). The average white man who is 75 may experience a single form of discrimination (ageism) and can still benefit from his other statuses (choice B is wrong). The average white woman who is 25 may experience a single form of discrimination (sexism) and can still benefit from her other statuses (choice D is wrong).

How might Anne Treisman's model account for the results in this research? A. Words and circles enter the central executive, go through the phonological loop, visuospatial sketchpad, and episodic buffer, and then into working memory. B. Words and circles enter a sensory store, go through a selective filter, and hit a bottleneck; only the words go through to higher level processing and then into working memory. C. Words and circles enter a sensory store, go through an attenuating filter, and hit a bottleneck where the "volume turns up" on the words and down on the circles, and the words get stored in working memory. Correct Answer D. Words and circles enter a sensory store, go through a phonological filter, hit a bottleneck, and then both words and circles reach working memory.

C. Treisman developed the attenuation model of selective attention. This theory would explain the research above in terms of the words and circles having entered the sensory store and gone through an attenuating filter. At that point they hit a bottleneck where the "volume turns up" on the words and down on the circles, so that the words can be stored in working memory and then recalled (choice C is correct). Words and circles entering a central executive into a phonological loop, a visuospatial sketchpad, and an episodic buffer before arriving at memory describes Baddeley's model of working memory (choice A is wrong). Words and circles going through a selective filter describes the selective filter theory of selective attention (choice B is wrong). A pathway in which words and circles enter a sensory store, proceed to a phonological filter, and then reach working memory is actually an amalgam of Baddeley's model and the selective filter model and does not exist (choice D is wrong).

Many forces influence how people relate to one another, become socialized beings, and learn to develop relationships. Which of the following is NOT one of the agents of socialization? A. Family B. School C. Travel Correct Answer D. Television

C. While travel can be highly educational in terms of introducing people to other cultures and ways of life, it is not specifically an agent of socialization; it does not necessarily help people learn how to relate, as one can travel alone and avoid contact with others (choice C is not an agent of socialization and is therefore correct). Family is one of the first and primary sources of socialization from which a person learns how to relate to others --- one's parents, siblings, and extended family (depending on the family structure). Lessons learned from the family often form the basis for many future relationship choices (choice A is an agent of socialization and is therefore wrong). School is another primary source of socialization, as teachers try to educate children about the values of the specific culture and the important skills needed to survive in that culture (choice B is an agent of socialization and is therefore wrong). Television, along with other forms of mass media, provide people with a glimpse of how to relate to the larger world, introducing people to other lifestyles as well as popular trends and ideas (choice D is also an agent of socialization and is therefore wrong).

Suppose that the researchers in Study 1 would like to factor in the possibility that students may have abandoned the wristbands out of boredom rather than because members of the Academic Dorm adopted them. Which of the following would be the most appropriate way to control for this possibility? A. Giving 50 wristbands to random students and faculty members at the college, and then measuring how many students from the Target Dorm notice non-Academic Dorm students or faculty wearing the wristbands. B. Giving 50 wristbands to students who attend the same classes as the Target Dorm and Academic Dorm residents, and then measuring how many of these students stop wearing the wristbands once they notice that members of the Academic Dorm also wear these wristbands. C. Giving 50 wristbands to a third dorm right next door to the Target Dorm and Academic Dorm (which also shares dining, recreational, and academic facilities with the Target Dorm and the Academic Dorm), and then measuring how many of these students notice the Target Dorm students and Academic Dorm students wearing the wristbands at the beginning and end of the trial period. D. Giving 50 wristbands to a third dorm that is far enough away from the Target Dorm and the Academic Dorm that students would not share dining, recreational, or academic facilities with students who reside there and would rarely, if ever, come into contact with them, and then measuring how many of the students from the third dorm still wear their wristbands at the end of the trial period. Correct Answer

D. A control group is a group of subjects who are not subjected to the actual experimental manipulations of the study in order to prevent any influence of the independent variable upon them, although they are the same as the other subjects in other critical respects. Thus, the use of a control group prevents other variables from confounding the results of the study. By distributing the same number of wristbands to a dorm that is separated from the Target Dorm and Academic Dorm, and then measuring the number of students from this group who still wear their wristbands at the end of the trial period, researchers should be able to determine a baseline level of abandonment of the wristbands based on boredom. Therefore, this would be the most appropriate way to control for boredom in Study 1 (choice D is correct). Randomly giving wristbands to students and faculty at the college and measuring how many students from the Target Dorm notice non-Academic Dorm students or faculty wearing the wristbands would not create an appropriate control group (because it contains students and faculty), nor would this measure help researchers understand anything about boredom (choice A is wrong). Distributing wristbands to classmates of the Academic and Target Dorm residents introduces a confounding variable because the fact that classmates are wearing the wristbands might affect the decisions of students in the Target Dorm to wear them; moreover, this would not help the researchers understand anything about boredom (choice B is wrong). Giving wristbands to a third dorm that shares dining, recreational, and academic facilities with the Target Dorm and Academic Dorm also introduces a confounding variable, and prevents students from this third dorm from serving as controls because it is highly probable that the control group will be influenced by the experimental variables (choice C is wrong).

According to ancient traditional Judaism, the first-born son must be redeemed through pidyon haben. This means that he will not be required to serve as a priest as outlined in holy texts. Accordingly, thirty days after the child is born (unless the date falls on Shabbat), his parents must redeem him by paying the modern equivalent of five silver shekels. Pidyon haben is an example of a: A. belief. B. value. C. norm. D. ritual. Correct Answer

D. A ritual is an action or ceremony that is performed consistently under prescribed circumstances. Pidyon haben takes place in the Jewish faith when a first son is born (prescribed circumstances); the exchange of money and the time at which the event occurs are also consistent from family to family. Pidyon haben, therefore, is a form of ritual (choice D is correct). A belief is something one concludes about the world. Many Jews subscribe to the belief that Pidyon haben is necessary; however, the event itself is not a belief (choice A is wrong). Values often derive from beliefs and correspond to ideas and concepts that individuals deem important. Honesty and integrity, for example, might be considered values by a population (choice B is wrong). A norm is a standard maintained by a society regarding how individuals should behave in particular situations. For instance, in a synagogue individuals would behave in accordance with established norms (choice C is wrong).

Which of the following does NOT directly help to explain why blacks have roughly 50% higher obesity rates than whites do in the United States? A. Due to racial residential segregation, neighborhoods that are predominantly black have limited access to safe outdoor recreational facilities. B. Due to racial residential segregation, impoverished neighborhoods have fewer grocery stores and more fast food restaurants. C. In poorer neighborhoods the public schools are underfunded and therefore provide lower-quality hot lunches. D. In poorer neighborhoods the public schools are underfunded, and therefore the school day is shorter than average, sometimes by as much as an hour. Correct Answer

D. A shorter school day would not directly explain why blacks have roughly 50% higher obesity rates than whites do in the U.S. (choice D is correct). If predominantly black neighborhoods have limited access to safe outdoor recreational facilities, this helps directly explain higher obesity rates for blacks (choice A is wrong). If impoverished neighborhoods have fewer grocery stores and more fast food restaurants, and the passage indicates that there is a higher percentage of blacks in poorer neighborhoods, then this helps directly explain higher obesity rates for blacks (choice B is wrong). If public schools are underfunded in poorer neighborhoods, providing lower-quality hot lunches, and the passage indicates that there is a higher percentage of blacks in poorer neighborhoods, this also helps to directly explain higher obesity rates for blacks (choice C is wrong).

Children with ADHD, Inattentive Type demonstrate functional deficits with working memory. If children with ADHD, Inattentive Type particularly struggle to remember information that they have heard, which of the following systems is most likely deficient? A. iconic sensory memory B. The episodic buffer C. The visuospatial sketchpad D. The phonological loop Correct Answer

D. According to Baddeley's model of working memory, there are three important systems that help process information; the phonological loop is the system that is responsible for allowing us to repeat auditory information back to ourselves long enough in working memory to either write the information down or to somehow encode it into long-term memory; deficits in the phonological loop would best explain why children with ADHD, Inattentive Type struggle with remembering auditory information (choice D is correct). The episodic buffer and visuospatial sketchpad are the two other systems in Baddeley's model; the episodic buffer integrates information across domains (choice B is wrong), while the visuospatial sketchpad is the site for processing visual and spatial information in working memory (choice C is wrong). Iconic sensory memory is not part of the working memory system, but rather the sensory memory system. Iconic sensory memory contains an extremely short-term (less than a second) storage capacity for visual information (choice A is wrong).

A school-aged boy has demonstrated an understanding of conservation, is focused on living up to the expectations of others, and tries to be a "good boy", but cannot yet reason about abstract moral concepts and does not yet feel a duty to uphold social rules and conventions. He is likely to be in which of the following stages, according to Piaget and Kohlberg, respectively? A. Preoperational thought; Stage 2 (self-interest orientation) B. Concrete operational thought; Stage 2 (self-interest orientation) C. Preoperational thought; Stage 3 (interpersonal accord and conformity) D. Concrete operational thought; Stage 3 (interpersonal accord and conformity) Correct Answer

D. According to Piaget, a child who has demonstrated a grasp of conservation but cannot yet reason about abstract moral concepts should fall within the concrete operational thought stage; children in the preoperational thought stage have not yet grasped the principle of conservation (choices A and C can be eliminated). According to Kohlberg, a child who tries to live up to the expectations of others and to be a "good boy", but does not yet feel a duty to uphold social rules and conventions, should fall within the third stage of moral reasoning (choice B is wrong and choice D is correct).

If a person is hit by a car in a hit-and-run accident on a busy metropolitan street in the middle of the day with many people around, the likelihood that someone will call for help will actually decrease in relation to the large number of people present because of which phenomenon? A. Self-serving bias B. Just-world phenomenon C. Conformity D. Bystander effect Correct Answer

D. According to the theory underlying the bystander effect, there is diffusion of responsibility among the individuals present during an emergency. Thus, most people assume, often mistakenly, that someone else will take the necessary action (such as calling the police or an ambulance). The likelihood that someone will take appropriate action is inversely correlated with the number of people at the scene (choice D is correct). A self-serving bias is the tendency of people to attribute their successes to themselves and their failures to external causes (choice A is wrong). The just-world phenomenon is the belief that the world is indeed fair and people get exactly what they deserve (choice B is wrong). Conformity involves adjusting one's behavior and attitudes in accordance with the behavior and attitudes of members of a reference group (choice C is wrong).

It has been proposed that there may be variations in the OXTR gene in individuals with a specific psychological disorder. In particular, researchers cite the OXTR gene's role in a variety of social behaviors, including trust, social comprehension, and attachment, as evidence that there may be differences between people with and without this particular disorder. Researchers likely believe that variation in the OXTR gene contributes most to which one of the following disorders? A. Schizophrenia B. Obsessive-compulsive disorder C. Attention deficit/hyperactivity disorder D. Autism spectrum disorders Correct Answer

D. Although research does suggest that the OXTR gene may be associated with general psychopathology, of the choices listed autism spectrum disorders are characterized to the greatest extent by deficits in social behaviors such as trust, social comprehension, and attachment. Research has suggested that autistic individuals may be more likely to exhibit an A allele (choice D is correct). Schizophrenia is often characterized by delusional thinking, hallucinations, and disorganized behavior, but social deficits are often only a byproduct of other psychotic symptoms (choice A is wrong). Obsessive-compulsive disorder is characterized by persistent and obsessive thoughts and repetitive behaviors, but is also not strongly linked to social deficits (choice B is wrong). Attention deficit/hyperactivity disorder (ADHD) is characterized by problems with maintaining attention and controlling impulsive behaviors, but is only tangentially linked to social deficits (choice C is wrong).

Which one of the following is LEAST likely to be the iatrogenic result of diagnostic labeling? A. Eating disorders B. Drug addiction C. Social deviance D. Alzheimer's disease Correct Answer

D. Alzheimer's disease is caused by neurodegeneration in the brain. Accordingly, someone who has the plaques and tangles characteristic of Alzheimer's disease is likely to experience symptoms of the illness irrespective of whether his or her self-image is tied to the disorder (choice D is correct). The other three choices all have a strong conduct component inherent in the diagnosis. Accordingly, one who is labeled as having an eating disorder may be more likely to engage is disordered eating behaviors (choice B can be eliminated), while one who is labeled as a drug addict may be more likely to feel helpless to control or limit drug use (choice C can be eliminated). Someone labeled as a social deviant is much more likely to engage in deviant behavior because of this label (choice C can be eliminated).

Which of the following reflects the researchers' attempt to minimize adaptation effects during the experiment? A. Using wild mice rather than domesticated miceYour Answer B. Using sugar water rather than pond water during the training stage C. Rearranging the bottles after a human subject detected sulfur D. Limiting the human subjects' inhalation time to two secondsCorrect Answer

D. An adaptation effect is the phenomenon by which an organism becomes accustomed to a sensory stimulus, which results in a higher threshold for detection of that stimulus. Since the experiment in this study was designed to measure detection thresholds, it was necessary to limit participants' inhalation time to two seconds so that they would not become accustomed to the smell of sulfur. Otherwise, their olfactory thresholds for the detection of sulfur may have measured higher than their actual levels (choice D is correct; choices A, B, and C are wrong).

An experiment is conducted that is designed to assess whether children are more likely to adopt a false memory of an event if they perceive that event as plausible or likely to have occurred. Using a technique similar to the "lost in the mall" method, one group of children is presented with a false memory of having choked briefly on a piece of food. A second group of children is presented with a false memory of having been taken on a vacation to a foreign country that the child in fact never visited. A third group is presented with a false memory of having been abducted and held captive for several days by a "bigfoot" type creature. In this case the plausibility of the false memories is a: A. confounding variable. B. participant variable. C. dependent variable. D. independent variable. Correct Answer

D. An independent variable is one that the researcher manipulates (choice D is correct). A confounding variable is one that the experimenter does not intentionally manipulate but which may vary with the independent variable and thus confuse the results of the study. Plausibility was being intentionally manipulated in this case (choice A is wrong). A participant variable is a type of extraneous (uncontrolled) variable that is related to the individual characteristics of the subjects. Plausibility is a characteristic of the false memories, not of the participants (choice B is wrong). The dependent variable of an experiment is that which is being measured, not manipulated (in this case the rate of false memory implantation is the dependent variable; choice C is wrong).

All of the following biological developmental milestones that change the brain occur during the adolescent years EXCEPT: A. Cell proliferation (particularly in the limbic system and prefrontal lobes) B. Myelination Your Answer C. Synaptic pruning (of unused or unnecessary connections) D. Codification of neural networks for basic motor skillsCorrect Answer

D. Neural network development for motor skills is part of early brain development and is essentially complete for basic motor skills (such as crawling, walking, jumping, and writing) before adolescence (choice D does not occur during adolescence and is therefore correct). Cell proliferation, particularly in the prefrontal lobes and in the limbic system, is one of the changes to the brain that occurs during adolescence, as the prefrontal cortex is required for abstract thinking (choice A is wrong). Myelination, the creation of myelin sheaths around neurons in the brain to strengthen connections, is another change that occurs during adolescence (choice B is wrong). Synaptic pruning, which clears the brain of unnecessary connections, is the third major developmental change in the brain during adolescence (choice C is wrong).

According to Piaget's theory of development, at which stage do children learn that things can be represented through words and images? A. Formal operational stage B. Sensorimotor stage C. Concrete operational stage D. Preoperational stage Correct Answer

D. During Piaget's preoperational thought stage (around ages 2 to 7), children learn that things can be represented through symbols, such as words and images; this is an important foundation for the development of more advanced language skills. At this stage children still lack logical reasoning abilities, but they begin to understand the world of representational symbols (choice D is correct). The formal operational stage occurs when a child reaches the approximate age of 12. This is when the child begins to use reason and to understand complex abstract thought and morality (choice A is wrong). The sensorimotor stage occurs during the first two years of life, during which a child explores his or her world with the five senses and through movement. This is also when children learn object permanence (choice B is wrong). The concrete operational stage lasts from about ages 7 to 12, during which the important lesson is conservation (i.e., that quantities remain the same despite changes in shape). This is also the age at which children begin to grasp abstract mathematical concepts (choice C is wrong).

The situation-profile of child 1 (Figure 1) best supports which of the following conclusions? A. Child 1 is a particularly hostile child. B. Child 1 is less hostile than the average child. C. Situation 5 is a situation that tends to elicit hostility from all children. D. Child 1 tends to exhibit less hostility in situation 2 than in situation 5. Correct Answer

D. Figure 1 shows that child 1 consistently displayed less hostility in situation 2 than in situation 5. This suggests that situation 5 is more likely to induce hostility in child 1 than is situation 2 (choice D is correct). Figure 1 alone does not support the conclusion that child 1 is more or less hostile than other children, as there is no other child with whom to compare child 1 (choice A and choice B are wrong). Child 1 displayed the most hostility in situation 5. However, on the basis of Figure 1 alone it is unclear whether situation 5 will induce hostility in all children; it is possible that another child may be less hostile in situation 5 than in another situation (choice C is wrong).

What conclusions can be drawn from the data presented in Figures 1 and 2? A. Hypnosis is more strongly correlated with effective pain reduction for male participants than for female participants. B. Hypnosis is more strongly correlated with effective pain reduction for female participants than for male participants. C. Hypnosis is not correlated with pain reduction for either male or female participants. D. Hypnosis is equally correlated with pain reduction for male and female participants. Correct Answer

D. Figures 1 and 2 indicate that, for both male and female participants, pain ratings dropped drastically from a 10-point rating in phase 1 all the way to a 1-point rating in phase 2; this shows that, in this particular experiment, hypnosis appeared to be equally effective for the two genders in terms of pain reduction (choice D is correct). Although male participants did start out phase 2 with an average pain rating approximately 1 point less than the female participants did, as the experiment continued the gender groups averaged out to a rating of approximately 1. Accordingly, it cannot be determined from these figures that one gender fared better than the other (choices A and B are wrong). Based on the data in these figures, it would appear that hypnosis was indeed effective in reducing pain for these particular participants, under these particular circumstances (choice C is wrong).

Prior to his development of the structural theory, Freud reported the existence of sexuality in children and discussed the way in which mature adult sexuality develops. Which of the following is the correct chronological order of the stages of psychosexual development? A. Oral, phallic, anal, latency, and genital stages B. Anal, oral, genital, latency, and phallic stages C. Genital, anal, oral, latency, and phallic stages D. Oral, anal, phallic, latency, and genital stagesCorrect Answer

D. Freud believed that biology is directly connected with human development, and therefore that the stages of psychosexual development correspond to primary biological needs. Thus, as small infants are focused primarily on eating as a source of pleasure, the oral stage accordingly comes first (choices B and C are wrong). The next stage in human development is the anal stage, as toddlers begin to find pleasure in the production of, or withholding of, bodily waste (choice A is wrong). Children next begin to focus on the differences between boys and girls, and male children begin to develop feelings of jealousy towards their fathers with regard to the relationship with the mother; this stage - the phallic stage - is the one in which the Oedipal complex occurs (choice D is correct). Next, children enter into the latency stage, in which sexual energy is redirected into forming same-sex friendships and developing the skills that will be necessary for later life, such as schoolwork. Finally, adolescents enter into the genital stage, which is the stage of mature adult sexuality.

Prevalence studies suggest that household composition influences the risk of child maltreatment. There is some evidence that the risk is higher for single-parent families, where there are significant differences in parenting methods when compared to two-parent families. Which of the following suggestions might explain these findings? I. Families with a single parent struggle with more economic challenges, such as less income potential. II. Families with a single parent struggle with more social challenges, such as social stigma. III. Families with a single parent struggle with more personal challenges, such as heightened stress.

D. Item I is true: It can be argued that there are more economic concerns in single-parent households (choice C is wrong). In these families, there is one less adult with an earning potential. Thus, the financial responsibilities of the single parent are higher, and experts suggest that this economic deprivation mediates the relationship between household composition and child maltreatment. This is more pronounced in studies of neglect, the most common form of child maltreatment. Item II is true: It can be argued that there are more social concerns in single-parent households (choice B is wrong). In these families, there is often the implication of divorce (even though it is also possible that the remaining parent was widowed). It is true that divorce is becoming more accepted, but in some parts of the world, it is still difficult for women in particular to support their families alone. Thus, these families face more social stigma, which could also contribute to the relationship between household composition and child maltreatment. Item III is true: It can be argued that there are more personal challenges in single-parent households (choice A is wrong and choice D is correct). In these families, the absence of the second parent requires the single parent to balance more responsibilities (e.g., the challenges of raising a child) without the support of a partner. Thus, the additional stress experienced could also contribute to the relationship between household composition and child maltreatment.

Which of the following statements is/are true regarding reproductive health in society? I. Community clinics have a manifest function of providing women's health resources to lower-income citizens. II. A teenager who becomes pregnant is more likely to have children who also end up having babies at a very young age; this is an example of social reproduction. III. Obtaining contraception and pregnancy prevention information is an issue of both accessibility and availability in most lower-income communities. A. I and II only B. I and III only Your Answer C. II and III only D. I, II, and III Correct Answer

D. Item I is true: a manifest function of a social institution it the intended purpose of that institution; providing women's health resources to lower-income citizens is the manifest function of community clinics, many of which provide free counseling and resources (choice C can be eliminated). Item II is true: social reproduction refers to the structures in society that transmit social inequality from one generation to the next Item. In other words, the lack of knowledge and access to resources which might have made it more likely for a teenager to become pregnant in the first place will also make it more difficult for that teenager to provide a better life to her children, which tends to perpetuate a cycle of poverty and makes it harder for the next generation to rise above their circumstances. Therefore, the fact that women who get pregnant at an early age are more likely to raise children who also end up having babies very young is an example of social reproduction (choice B can be eliminated). Item III is true: most lower-income neighborhoods do not have as many healthcare resources (both contraception and information) available to them. For example, fewer doctors tend to practice in poorer communities, schools may not have the resources to provide sexual education, and community clinics may not have the fund for outreach or free condoms. These are all issues of availability (the needed resources are not available). Similarly, even when resources do exist, there are also issues of accessibility that also prevent individuals in lower-income communities from obtaining the information or resources they need. For example, even if a low-cost clinic is in the neighborhood, it might be so overbooked that it is difficult to get an appointment when needed. Therefore, obtaining contraception and pregnancy prevention information is an issue of both accessibility and availability in most lower-income communities (choice A can be eliminated; choice D is correct).

Jax, a gay teenager, was asked to leave his parents' house when he came out as gay. Now he lives with a community of other teenagers in a youth center and he considers this community to be his family. Which of the following specific types of groups is this community for Jax? I. An in-group II. A primary group III. A reference group A. I only B. I and II only Your Answer C. II and III only D. I, II, and III Correct Answer

D. Item I is true: an in-group is a group that an individual belongs to and with which he or she identifies as a member (choice C can be eliminated). Item II is true: a primary group serves expressive functions, such as meeting emotional needs. A family is an example of a primary group and Jax considers his teenage friends to be his family (choice A can be eliminated). Item III is true: a reference group is a group that one might compare oneself to based on the fact that all members share a common identity. Jax can compare himself to his teenage friends because they are all in a similar situation—being young and living primarily in shelters (choice B can be eliminated and choice D is correct).

Studies have shown that individuals with a larger-than-average rostrum, a part of the corpus callosum that is responsible for attention and focus, are more susceptible to hypnosis than are those with rostrums that are average or below average in size. The corpus callosum: I. is composed entirely of white matter. II. connects the left and right cerebral hemispheres and is responsible for interhemispheric communication. III. can be surgically severed in patients who suffer from severe seizures; severing this structure prevents seizures that originate in one hemisphere from affecting the other.

D. Item I is true: the corpus callosum is composed entirely of myelinated axonal projections extending from one side of the brain to the other; it is therefore composed entirely of white matter (choice C can be eliminated). Item II is true: the corpus callosum connects the two cerebral hemispheres and is responsible for interhemispheric communication (choice A can be eliminated). Item III is true: for patients experiencing very severe seizures (caused by inappropriate electrical activity in the brain) that originate on one side of the brain and spread to the other, severing the corpus callosum reduces the severity of the seizures; the electrical activity no longer has a route for spreading from the hemisphere of seizure origination to the other hemisphere. People who have undergone this surgery are known as "split-brain patients," and much has been learned about the lateralization of brain functions through studies on these unique patients (choice B can be eliminated and choice D is correct).

Which of the following statements demonstrates the idea that mass media are agents of socialization? I. Young people's body image is affected by the bodies depicted in magazines II. News media coverage of sexual assault focuses primarily on the victim III. Social media has become a pervasive aspect of everyday life A. I only B. I and II only C. II and III only D. I, II, and III Correct Answer

D. Item I is true: the mass media present and perpetuate a body image that is glorified by society. Exposure to these glorified images can influence young people to take on these body standards as ideal, and this is a form of socialization (choice C can be eliminated). Item II is true: when news media coverage focuses primarily on the victim of a sexual assault, women learn that it is their job to protect themselves from such assaults, but perpetrators do not learn to stop assaulting women. Emphasizing women's victimhood in mass media is a form of gender socialization that carries with it messages about whom to associate with, how much to drink, and how to dress oneself (choice A can be eliminated). Item III is true: social media can be considered mass media, and because of its pervasiveness, social media has become a powerful socializing force in almost everyones' lives, and has significantly impacted: how people communicate and interact, the types of information individuals and organizations share, how often information is shared, the lexicon, how business is conducted, and other such major phenomenia, both big and small (choice D is correct; choice B can be eliminated).

Microsociologists are interested in the mechanisms through which individuals shape their social realities. Researchers are able to answer these questions with an exclusive focus on micro-level factors using all of the following theoretical paradigms EXCEPT: A. constructionism. B. interactionism. C. the dramaturgical perspective. D. the feminist perspective. Correct Answer

D. Microsociologists focus on the effects of individual interactions, like the social meanings created through these interactions. Each of the theories listed considers micro-level social factors. However, the feminist perspective is not limited to consideration of the individual; this perspective does not place an exclusive focus on micro factors and choice D is therefore the correct answer. The feminist sociological approach examines the differences in the social experiences of men and women, thus considering micro-level questions, such as how gender is constructed through these interactions. The feminist perspective, however, is also concerned with the social structures that perpetuate gender inequalities in modern societies, such as the institution of marriage.

A child hits her brother and is punished by being sent to her room with no dinner. When she is about to strike her brother again the next day, she remembers how hungry she was the day before and stops herself. What kind of conditioning occurred here? A. Classical conditioning; her hunger is a biological reaction B. Classical conditioning; her hunger is involuntary C. Operant conditioning; thinking about hitting her brother makes her hungry Your Answer D. Operant conditioning; her reaction is voluntaryCorrect Answer

D. One of the major differences between classical conditioning and operant conditioning is whether the change in behavior is voluntary or involuntary. In classical conditioning, a natural (unconditioned) stimulus gets paired with a conditioned stimulus so that the conditioned stimulus eventually causes the same type of reaction that the unconditioned stimulus does. In contrast, operant conditioning is about the subject's choosing to perform, or not to perform, a particular action because of the consequences that previously resulted from that action. In this case, the child is making a conscious decision not to hit her brother because of the negative punishment (lack of dinner) that occurred when she hit him yesterday; therefore, her parents are using operant conditioning to make her change her behavior (choice D is correct). Although the child's hunger is a biological reaction to not eating dinner, and is theoretically an unconditioned response, her parents are not attempting to make hunger a conditioned response, and therefore they are not engaging in classical conditioning (choice A is wrong). Although her hunger is involuntary, it is the child's decision not to hit her brother that is the focus, making the issue of whether or not her hunger reaction is voluntary irrelevant (choice B is wrong). If thinking about hitting her brother made her hungry, which it would not, this would be a situation involving classical conditioning, not operant conditioning (choice C is wrong).

Members of the traditional immigrant families studied might have been drawn to the Netherlands due to pull factors, such as: A. drought and subsequent crop failure. B. political rights in a safe environment. C. insufficient resources and services. D. better prospects and earning potential. Correct Answer

D. Patterns of migration are influenced through both push and pull factors. The push factors are those characteristics of an area that "push" people to emigrate from it (where emigrate means to leave an area); the pull factors are those characteristics of an area that "pull" people to immigrate to it (where immigrate means to enter an area). These push and pull considerations can be economic, political, or social in nature. The question is concerned with pull factors, which in this situation would be the positive attributes of the Netherlands as a new potential home. The negative attributes of the countries from which migrants emigrated are not of concern; thus, the push factors listed can be eliminated (choice A and C are wrong). The question is also interested in the reasons for traditional immigration. The first paragraph of the passage explains that traditional immigrants entered the Netherlands as labor migrants or as a result of previous colonization while non-traditional immigrants entered as refugees. Thus, the expected pull factor for traditional migration involves economic concerns, like increased earning potential in the Netherlands (choice D is correct). The second pull factor, political rights, is more expected to contribute to non-traditional migration (choice B is wrong).

Which of the following is NOT a cluster B personality disorder? A. Antisocial personality disorder B. Borderline personality disorder C. Histrionic personality disorder D. Paranoid personality disorder Correct Answer

D. Personality disorders are grouped based on similarities in their characteristics. Cluster B disorders are characterized by heightened emotional arousal, as well as by dramatic, erratic, or impulsive behavior. Paranoid personality disorder is characterized essentially by distrustful behavior; it is a cluster A (odd) disorder and not a cluster B (dramatic) disorder (choice D is not a cluster B disorder and is therefore correct). Antisocial, borderline, and histrionic personality disorders are all characterized by poor emotional regulation and are cluster B disorders (choices A, B, and C are wrong).

Which of the following neurochemical changes would increase the likelihood that a visual signal will be transmitted to the brain? A. An increase in post-synaptic glutamate released by the photoreceptors B. An efflux of Cl- ions from the photoreceptors C. An efflux of K+ ions from the bipolar cells D. Closure of Na+/Ca2+ cyclic nucleotide-gated (CNG) channels in the photoreceptors, which are normally open during rest (in the dark) Correct Answer

D. Photoreceptors are depolarized during rest (in the dark) due to the constant influx of positive ions through the CNG channels. Thus, in order to increase the likelihood that a visual signal will be transmitted to the brain (i.e., to make it more likely that a visual stimulus of light will be perceived as an image), these photoreceptors must be hyperpolarized. During rest (in the absence of light), photoreceptors are depolarized, constantly releasing glutamate, which inhibits the bipolar cells from depolarizing. For a visual signal to be transmitted to the brain, the photoreceptor must be hyperpolarized, which inhibits the release of glutamate, allowing the bipolar cells to depolarize. Closure of CNG channels in the photoreceptors would prevent positively-charged ions from continuing to enter the photoreceptor, and would therefore make the membrane potential more negative, or hyperpolarized; this would increase the likelihood that a visual signal will be transmitted to the brain (choice D is correct). An increase in post-synaptic glutamate released by the photoreceptors would decrease the likelihood that a visual signal will be transmitted to the brain because glutamate synapses on bipolar cells and inhibits them, as described in the first paragraph (choice A is wrong). An efflux of Cl- ions from the photoreceptors would further depolarize them; they would become more positively charged as a result of the loss of negatively charged Cl-, which would decrease the likelihood that a visual signal will be transmitted to the brain (choice B is wrong). An efflux of K+ ions from the bipolar cells would result in more negatively charged (or hyperpolarized) bipolar cells, making them less likely to fire, and decreasing the likelihood that a visual signal will be transmitted to the brain (choice C is wrong).

Later in her interview, Subject 103 talked about her experience with smoking. She mentioned that she was interested in quitting, but that stopping smoking made her feel irritable and stressed. In this case, irritability is one sign of: A. psychological dependence. Your Answer B. biofeedback. C. physiological arousal. D. physical dependence. Correct Answer

D. Physical dependence is evidenced by symptoms of withdrawal that are calmed by further use of the addictive substance. In this case Subject 103 experiences irritability, but this is alleviated by the nicotine that she gets from smoking (choice D is correct). Psychological dependence can result from using a drug to cope with painful emotions. Although Subject 103 experiences painful emotions because she does not have a community that accepts her identity, the information in the question stem deals with her physical symptoms of dependence and not with the potential underlying psychological causes (choice A is wrong). Biofeedback is a therapeutic technique in which individuals are given information about their autonomic responses to stressful situations and taught to use this information to control their responses (choice B is wrong). Physiological arousal is an excitation of the body's internal state; it is an aspect of emotional response (choice D is wrong).

Psychological drug dependence is characterized by: A. tolerance, wherein higher and higher quantities of the drug are necessary to achieve the desired effect. Your Answer B. downregulation of neurotransmitter activity in specific brain regions, such as the nucleus accumbens, in response to prolonged use of the drug. C. severe physical withdrawal symptoms upon cessation of drug use. D. the use or abuse of the drug in response to painful emotional triggers like depression or anxiety. Correct Answer

D. Psychological drug dependence is characterized by the use or abuse of the drug in response to painful emotional triggers, such as depression or anxiety (choice D is correct). Psychological and physiological drug dependence are similar aspects of drug addiction; physiological dependence is marked by a physical "craving" for the drug, while psychological dependence is a "need" for a particular drug because it causes enjoyable mental effects (the two forms of dependence are often both present in drug addiction). However, psychological dependence is not characterized by tolerance (choice A is wrong), downregulation of neurotransmitter activity in the nucleus accumbens in response to prolonged use (choice B is wrong), or symptoms of withdrawal (choice C is wrong); these are all physiological phenomena associated with drug dependence.

Robert Merton proposed the concept of unintended consequences, which he defined as unintended outcomes of purposeful social action. This suggests the importance of routinely evaluating models of social intervention, such as global health initiatives, for unintended consequences. For example, during smallpox eradication efforts in India coercive vaccination policies caused many individuals' bodies to become resistant to antibiotics. In this case, the resultant drug resistance is best described as which of the following? A. A manifest function B. A latent function C. A manifest dysfunction D. A latent dysfunction Correct Answer

D. Robert Merton is the theorist credited with the distinction between manifest and latent functions and dysfunctions. In the global health situation described, the reported drug resistance among individuals in India is an example of a latent consequence of social intervention (choices A and C are wrong). Latent functions and dysfunctions are the unrecognized and unintended consequences of social patterns; manifest functions and dysfunctions are the recognized and intended consequences of social patterns. Furthermore, this situation is a social dysfunction, rather than a function, because it is an undesirable consequence that can reduce social stability (choice B is wrong and choice D is therefore correct).

If a bisexual female reports feeling social pressure to be either lesbian or straight, which causes her significant stress, what term best describes the stress this subject is feeling about her social role? A. Role conflict Your Answer B. Master status C. Role exit D. Role strain Correct Answer

D. Role strain is when having a single status results in conflicting expectations; if a bisexual female reports feeling social pressure to be either lesbian or straight, which causes her significant stress, she is experiencing conflicting expectations about her bisexual identity (choice D is correct). Role conflict happens when there are conflicting societal expectations for multiple statuses held by the same person. For example, a male kindergarten teacher—societal expectations for being a man and being a kindergarten teacher easily come into conflict (choice A is wrong). Master status is when one part of someone's identity comes to dominate their identity, to the exclusion of other parts of their identity (choice B is wrong). Role exit is the process of disengaging from one role in order to take up another. An example is the process of disengaging from a "pre-med" identity to take on a "medical student" identity (choice C is wrong).

The myelin sheath is a dielectric that surrounds axonal membranes and facilitates electrical transmissions of neuronal signals. Demyelination is implicated in the exacerbation of Alzheimer's symptoms. Impairment of cognitive abilities due to demyelination most likely occurs because: A. increased speed of processing results in improved episodic memory. B. decreased speed of processing results in improved semantic memory. C. decreased speed of processing results in improved episodic memory. D. decreased speed of processing results in impaired semantic memory. Correct Answer

D. Semantic memory refers to the memory of meanings and other concept-based knowledge. As demyelination occurs, neuronal signals that carry meanings slow down, damaging the memory of them (choice D is correct). Episodic memory is the ability to contextualize the memory of autobiographical events; increased speed of neuronal processing could result in improved episodic memory, but this would not impair cognitive abilities and is not what the question stem asks (choice A is wrong). Decreased speed of neuronal processing would not result in memory improvement, whether semantic or episodic (choices B and C are wrong).

Which of the following is NOT a theory that accounts for gender differences? A. Gender stereotyping B. Gender role preference C. Gender schema theory D. Sex drive Correct Answer

D. Sex drive in humans is a function of physiological (e.g., hormonal) and non-physiological factors, but has nothing to do with gender differences (choice D is not a theory that accounts for gender differences and is therefore correct). Gender stereotyping describes the all-pervasive ideas in a culture to the effect that certain types of clothes, comportment, toys, colors, etc. are appropriate for only one gender (choice A accounts for gender differences and is therefore wrong). Gender role preference involves one's decision that, despite having the physiological and biological characteristics of one gender, he or she identifies with the other gender and proceeds to comport him or her self accordingly (choice B accounts for gender differences and is therefore wrong). Gender schema theory explains how gender differences are transmitted; the theory incorporates gender stereotypes and biological characteristics (choice C accounts for gender differences and is therefore wrong).

The passage indicates that women are tokens in medical and law schools. Which of the following concepts is a possible effect of tokenism? A. Group polarization B. Groupthink C. Social facilitation D. Deindividuation Correct Answer

D. The passage defines tokens as "members of a minority group representing the whole group, rather than individuals representing only themselves". Deindividuation is a loss of self-awareness in a social group in exchange for identification with that group. Because women are a minority group, they are at risk of losing self-awareness because they are seen as representing the whole group rather than as individual people (choice D is correct). Group polarization involves like-minded individuals becoming more extreme in their acts and decisions once they are in a group. Since tokenism is a concept that is projected onto women in professional schools, group polarization is unlikely as an effect (choice A is wrong). Groupthink occurs when members of a group are apt to censor and discourage dissenting viewpoints in order to maintain group harmony and cohesion (choice B is wrong). Social facilitation happens when an individual's performance improves because he or she is part of a group (choice C is wrong).

Children learn their roles in the world from their parents and other social influences, and roles and rules vary across families and cultures. For instance, spitting may be deemed offensive in one household but acceptable in another. Specific norms are therefore created by individuals and would not exist in the absence of human beings. This idea that people develop meanings of the world and thereby agree upon values is grounded in the theory of: A. symbolic interactionism. B. functionalism. C. conflict theory. D. social constructionism. Correct Answer

D. Social constructionism is concerned with the ways in which people understand, interpret, and give meaning to the world. It explains how human beings socially construct such aspects of life as roles and rules. Therefore, the same action, such as spitting, can be interpreted differently by different groups of people. The taboo, however, would not exist if human beings had not created it in the first place (choice D is correct). Symbolic interactionism studies small-scale interactions between and among human beings. It asserts that people act based upon the meanings that they have assigned to objects, but such meanings are fluid and subject to change. While symbolic interactionism is very similar to social constructionism, it is important to note that only the former focuses on the way people act (choice A is wrong). Functionalism involves multiple parts of an entire unit or society working together to establish equilibrium (choice B is wrong). Conflict theory refers to the ways in which societies transform over time. Often two opposing viewpoints arise, and constant competition leads to an ever-changing world. This perspective also focuses on social control and power struggles (choice C is wrong).

A high school student has recently been moved into a remedial English class due to failing grades. Since then she is often ridiculed by other students who call her "stupid," and her honor society friends tend to avoid her while at school. This is an example of: A. a self-fulfilling prophecy. B. stereotype threat. C. discrimination. Your Answer D. stigma. Correct Answer

D. Stigma involves a negative reaction to an individual or group based upon some perceived characteristic or status that is viewed as inferior or is otherwise degraded. The reaction of this student's peers strongly suggests that she has been stigmatized (choice D is correct). A self-fulfilling prophesy occurs when an expectation is ultimately realized because the expectation itself influences attitudes and behavior in such a way as to bring about its own fulfillment. In this case, there is no evidence that an expectation that she would fail English in fact caused her to do so. The negativity that she encountered from her peers occurred after she earned failing grades and was consequently transferred to the remedial class (choice A is wrong). Stereotype threat refers to the risk that performance may suffer when an individual fears that he or she will confirm a negative stereotype about his or her race, ethnicity, gender, etc. There is nothing to indicate that this student is a member of any group that is negatively stereotyped in terms of academic performance in English class (there is in fact a popular perception in the U.S. that females do better in English than their male counterparts). Moreover, there is no indication that this student's grades have been adversely affected by anxiety due to any stereotype (choice B is wrong). Discrimination occurs when individuals are unfairly denied rights or privileges, usually based upon immutable characteristics, like race, gender, or sexual orientation. There is no evidence that this student was denied any such rights or privileges in this scenario, and grades are not an immutable characteristic (choice C is wrong).

One year after the first administration the children are brought back to do the marshmallow task again, and the scores from the two administrations are significantly positively correlated. What does this demonstrate? A. Construct validity B. Inter-rater reliability C. Split-half reliability D. Test-retest reliability Correct Answer

D. Test-retest reliability is a test's ability to produce the same results over time (choice D is correct). Construct validity concerns how well a test measures what it is expected to measure, which is not demonstrated by retesting the same children a year later (choice A is wrong). Inter-rater reliability measures how well different raters match in their assessments, which is also not demonstrated by retesting the same children (choice B is wrong). Split-half reliability involves a multiple-item instrument and how well parts of it assess what is being measured compared to the whole; this is also not demonstrated by retesting the same children (choice C is wrong).

The hippocampus is a subpart of which brain structure? A. Executive functioning B. Basal ganglia C. Occipital lobe D. Limbic system Correct Answer

D. The hippocampus is part of the limbic system of the brain; a system which is responsible for many aspects of human functioning, including emotions and memory (choice D is correct). Executive functioning is generally thought to be regulated by the frontal lobes, which control inhibition and attention (choice A is wrong). The basal ganglia is situated at the base of the forebrain and is mainly comprised of striatum, the globus pallidus, the substantia nigra, the nucleus accumbens, and the subthalamic nucleus. It is associated with voluntary movement, procedural learning, and emotions, but is not associated with episodic memory as the hippocampus is (choice B is wrong). The occipital lobe of the brain is at the back of the head, and is primarily associated with vision (choice C is wrong).

Do the results of the study support both of the researchers' hypotheses? A. The results support both hypotheses because Figure 1 shows that students experienced lower amounts of stress due to interpersonal relationships than due to structural factors. B. The results support both hypotheses because Figure 1 shows that women reported more stress from sexism than did men. C. The results do not support both hypotheses because Figure 1 shows that men reported more stress due to racism than did women. D. The results do not support both hypotheses because Figure 1 shows that students experienced higher amounts of stress due to structural factors than due to interpersonal relationships. Correct Answer

D. The hypotheses mentioned in the passage are that "most students' stress would be shown to originate more from interpersonal relationships than from environmental and structural factors" and that "more women than men would report stress from gender discrimination because these professions are male-dominated". While the second hypothesis is borne out by the data (women faced more stress than men due to sexism), Figure 1 also shows that structural factors (exams, deadlines, and shortage of time) caused students more stress than did interpersonal relationships with faculty, spouses, and peers. Thus, the figure does not support both hypotheses (choice D is the correct answer and choices A and B are wrong). Gender discrimination is equivalent to sexism, not racism; racism is unrelated to the study's hypotheses (choice C is wrong).

Gentrification is best described as an example of which form of demographic change? A. Urban blight B. Urban decline C. Urban growth D. Urban renewal Correct Answer

D. The process through which deteriorated urban areas are rebuilt and become more upscale, called gentrification, is most often the result of urban renewal efforts (choice D is correct). Such efforts call for the redevelopment of urban areas, and gentrification, in particular, suggests a shift in the communities through economic developments that attract wealthier businesses and therefore wealthier residents. This is similar to the concept of urban growth, but gentrification is specific to the revitalization of existing urban centers; urban growth is a more ambiguous concept and studies of urban growth are most often concerned with the initial growth of cities as a result of historic industrialization (choice C is wrong). The passage states that gentrification is the opposite of urban decline, as it concerns the process through which people reenter cities (choice B is wrong). Urban blight is one of the results of urban decline in which cities that were once functioning rot to a state of disrepair; this is exemplified through "ruins photography" that features desolate landscapes (choice A is wrong).

The resource model of attention suggests that: A. human beings must "shine a spotlight" on the particular resource to which they wish to attend in order to focus their full attention. B. human beings have the resources to fill in any missing information and thus do not need to attend to everything. C. human beings selectively filter out unnecessary information according to their resources. D. human beings have a limited amount of resources for attention when performing tasks. Correct Answer

D. The resource model of attention states that human beings have a limited amount of resources for attention. Thus, if one wants to multitask, and the attentional needs exceed available resources, these tasks cannot be completed simultaneously (choice D is correct). The spotlight theory of attention is related more to visual attention and is concerned with where one focuses attention in one's visual field, as opposed to where one moves his or her eyes (choice A is wrong). That human beings fill in missing information is related to the priming effect; when people are "primed" or exposed repeatedly to particular sensory data, they are more likely to notice it. For example, if one is primed to be aware of monsters, he or she may be more likely to fill in the blank with "ster" when presented with "mon----" (choice B is wrong). The idea that human beings selectively filter out unnecessary information is part of the Broadbent filter model of selective attention, which states that sensory information enters a buffer and is then filtered based on various characteristics. Some sensory input decays while other data moves on to short-term and even long-term memory (choice C is wrong).

It can be concluded from Figure 1 that: A. the DA group scored significantly better than the FA group on Target Words. B. the FA group performed significantly better on Distractor Words than on Target Words. C. the FA group seems to demonstrate an "attentional boost," but there is no information regarding statistical significance. D. the DA group seems to demonstrate an "attentional boost," but there is no information regarding statistical significance.Correct Answer

D. The second paragraph states: " the first research showed that the images encoded with the white squares were remembered better, which is called an attentional boost effect, despite the divided attention situation." The attentional boost effect occurs when something this is meant to distract actually helps an individual remember. Figure 1 demonstrates that the divided attention (DA) conditions remembered both Target Words (65%) and Distractor Words (60%) better that the full attention (FA) conditions (45% for Target Words and 55% for Distractor Words). Therefore, Figure 1 does appear to indicate that the DA group is demonstrating an "attentional boost," but there is no information, either in the figure or the passage, regarding statistical significance, so it not possible to determine if the DA group performed significantly better than the FA group (choice D is correct; choices A and C and wrong). Since there is no information, in the figure or passage, regarding statistical significance, it is also not possible to conclude that the FA group performed significantly better on Distractor Words than on Target Words (choice B is wrong).

Suppose Matthew is the third of five children. For as long as he can remember, he has heard his parents refer to him as the "problem child". In turn, he often behaves inappropriately at home and in the classroom. This is an example of: A. attribution. B. discrimination. C. peer pressure. D. self-fulfilling prophecy. Correct Ans

D. The self-fulfilling prophecy occurs when the perceiver has certain expectations about the other and elicits those expectations accordingly; the individual acts in a way that confirms the perceiver's expectations. As described in the question stem, Matthew's parents believe that he is problematic and discuss this belief when he is within earshot of their conversations. Accordingly, Matthew misbehaves and confirms their expectations (choice D is correct). Attribution occurs when an individual draws an inference about how another behaves based on superficial evidence. For instance, someone might assume that a woman is overweight because she is lazy when, in reality, she has a thyroid problem (choice A is wrong). Discrimination is the act of treating a person or group unfairly based on perceived differences. The question stem does not indicate that Matthew is being treated differently or unfairly in this way (choice B is wrong). Peer pressure occurs when one feels pressured by his or her peers to act a certain way. While Matthew is expected to behave badly, he is not necessarily being pressured into doing so (choice C is wrong).

A forty-year-old woman is admitted to the emergency room after a serious car accident. Hospital staff attempt to obtain the woman's emergency contact information in order to have someone come to the hospital as a source of support. The woman refuses to provide any contact information, stating that she does not need, want, or have anyone whom she wishes to be contacted in her behalf. The woman further adds that she prefers to live a solitary life that is not dependent on others. Based on this limited information, which of the following best describes her interactions with her primary caregivers during childhood and her resulting attachment style? A. A secure attachment style resulting from interactions with loving and supportive parents who were available and comforting when needed B. A disorganized attachment style resulting from interactions with parents who were sometimes positive and, at other times, harsh and rejecting C. An anxious-resistant insecure attachment style resulting from interactions between parent and child that left the child feeling uncertain about the availability of the parent during times of need D. An anxious-avoidant insecure attachment style resulting from rejection of the child by the parent during times of nee

D. The woman's reluctance to form attachments to others and preference for a solitary life may indicate that she formed an anxious-avoidant insecure attachment style during childhood. This style is the result of the primary caregiver constantly rejecting or rebuffing the child when he or she sought comfort or protection. Because of these repeated rejections, the child comes to expect the same from other people. At the extreme, these individuals prefer to live a life free of any attachments to others, much like the woman described above (choice D is correct). If the woman had established a secure attachment style during childhood, she would likely have numerous individuals with whom she has bonded and formed strong relationships. Thus, she would have individuals in her life from whom she would like to receive comfort and support (choice A is wrong). This woman does appear to have difficulty trusting others, which could be indicative of a disorganized attachment. However, disorganized attachment usually does not lead to disavowal of relationships and significant connections to others. Rather, the quest to form interpersonal connections is hampered by poor social skills and an inability to regulate emotions (choice B is wrong). An anxious-resistant insecure attachment style usually results from parents who are sometimes supportive and comforting, and at other times absent and unavailable, leaving the child uncertain about the parent's availability. This leads to preoccupation in adulthood with being abandoned by significant others and to hesitation in forming relationships; when these individuals do form relationships they may display clingy behaviors. With this attachment style, the woman in question would likely be very motivated to have significant people in her life available during such a time of need (choice C is wrong).

Estimates from the National Office for Post Abortion Recovery and Healing suggest that around 70% of romantic relationships end after the procedure. Experts have provided much evidence that stress impacts our relationships. The dissolution of relationships post-abortion might be the result of which form of stress? A. An ambient stressor B. A catastrophe C. A daily hassle D. A significant life event Correct Answer

D. There exist three main categories of stressors from the most to least severe: catastrophes, significant life changes, and daily hassles (choice A is wrong). The post-procedure negative emotions resulting from an abortion might be attributed to the fact that the procedure suggests multiple significant life changes in a short period of time (choice D is correct). These major life events can be positive or negative, but their occurrence is rare enough to cause significant amounts of stress. There is associated acute stress, but chronic stress is also a possible effect. The process of an abortion suggests at least two significant life changes: (1) the pregnancy and (2) the abortion. There could be other considerations as well, such as conflict with close relations as a result of the pregnancy and/or the abortion, subsequent grief, etc. Though the experience is often a difficult one, it is not considered a catastrophe because these catastrophes are large-scale events that tend to have widespread effects on the societies experiencing them, such as in the case of natural disasters (choice B is wrong). Abortion is also not considered a daily hassle; these stressors are regular irritations that might have different effects on different people, such as managing finances (choice C is wrong).

An individual reports experiencing chronic stress because she cannot afford clothes suitable for job interviews. What does this individual lack as a result of her economic situation? A. Social capital B. Social reproduction C. Symbolic capital Your Answer D. Cultural capital Correct Answer

D. This individual lacks a cultural asset that would increase her likelihood of attaining employment; cultural capital refers to certain non-financial assets that promote or allow upward mobility in society, such as education, social graces, proper speech, specialized attire, etc. (choice D is correct). Social capital refers to the benefits of belonging to social networks; the question stem mentions appropriate attire for job interviews rather than interpersonal connections (choice A is wrong). Social reproduction refers to the processes through which social, cultural, and financial capital are transmitted generationally; the question stem does not mention interaction between generations (choice B is wrong). Symbolic capital refers to certain intangible assets, such as prestige, honor, and recognition, that help one advance within a society (choice C is wrong).

Which one of the following, if true, is inconsistent with the underlying principles of traditional groupthink theory as outlined in the passage? A. Most decisions made by cohesive groups do not result in groupthink. Your Answer B. The majority of faulty decisions caused by a groupthink process result in little or no significant harm. C. Groups that lack cohesiveness have much more difficulty arriving at a consensus than do cohesive groups. D. The higher the level of cohesiveness in a group, the better and more well-reasoned the decisions made by that group.Correct Answer

D. Traditional groupthink theory holds that member cohesiveness is the greatest cause of groupthink. If the quality of the decision-making process increases as cohesiveness increases, the latter does not cause groupthink (choice D is inconsistent with groupthink principles and is correct). The fact that most decisions made by cohesive groups do not reflect a groupthink process is not inconsistent with the theory. The more cohesive the group, the greater the probability of a groupthink-generated decision (choice A is not inconsistent with groupthink principles and can be eliminated). The extent of harm caused by groupthink is not pertinent to the fundamental principles of the theory as outlined above (choice B is not inconsistent with groupthink principles and can be eliminated). The fact that non-cohesive groups may have a harder time arriving at a consensus does not speak to the quality of the decisions that are ultimately reached by cohesive versus non-cohesive groups (choice C is not inconsistent with groupthink principles and can be eliminated).

Which of the following helps to explain why a single candle burning in a dark room is very noticeable to the human eye, but a single candle burning in a well-lit room does not produce any noticeable difference in brightness? A. Top-down processing B. The law of figure-ground perception C. Parallel processing D. Weber's Law Correct Answer

D. Weber's Law states that the just-noticeable difference between two light stimuli is proportional to the magnitude of the stimuli - or, an increment is judged relative to the previous intensity of light. In other words, for the difference in two light stimuli to be perceptible, they must differ by a constant proportion, not a constant amount. This law explains why a lit candle in two different rooms (one dark, one well-lit) will be perceived quite differently by the human eye and brain (choice D is correct). Top-down processing explains how experience and expectations influence our visual perception, but does not explain why a candle in two different rooms produces brightness that is perceived differently (choice A is wrong). Gestalt laws of visual perception indicate that distinguishing the relationship between figure and ground is important. However, Gestalt laws of perception in general do not explain the difference in brightness of a single visual stimulus in two different settings (choice B is wrong). Parallel processing describes the brain's ability to simultaneously process incoming stimuli of differing quality, but this does not explain the difference in brightness of a single visual stimulius in two different settings (choice C is wrong).


Conjuntos de estudio relacionados

IL Life - Provisions, Riders and Options combined sets

View Set

Focus on Vocabulary - Why are you happy?

View Set

NURS417 Ch10: Therapeutic Communication and Relationships

View Set

APES Unit 3 Quizlet Based on Resources

View Set

Sport Finance Final Exam Questions

View Set